Programs & Examples On #Sql server

Microsoft SQL Server is a relational database management system (RDBMS). Use this tag for all SQL Server editions including Compact, Express, Azure, Fast-track, APS (formerly PDW) and Azure SQL DW. Do not use this tag for other types of DBMS (MySQL, PostgreSQL, Oracle etc.). Do not use this tag for issues on software and mobile development, unless it is directly related to the database.

Using DateTime in a SqlParameter for Stored Procedure, format error

If you use Microsoft.ApplicationBlocks.Data it'll make calling your sprocs a single line

SqlHelper.ExecuteNonQuery(ConnectionString, "SprocName", DOB)

Oh and I think casperOne is correct...if you want to ensure the correct datetime over multiple timezones then simply convert the value to UTC before you send the value to SQL Server

SqlHelper.ExecuteNonQuery(ConnectionString, "SprocName", DOB.ToUniversalTime())

SQL Server dynamic PIVOT query?

Updated version for SQL Server 2017 using STRING_AGG function to construct the pivot column list:

create table temp
(
    date datetime,
    category varchar(3),
    amount money
);

insert into temp values ('20120101', 'ABC', 1000.00);
insert into temp values ('20120201', 'DEF', 500.00);
insert into temp values ('20120201', 'GHI', 800.00);
insert into temp values ('20120210', 'DEF', 700.00);
insert into temp values ('20120301', 'ABC', 1100.00);


DECLARE @cols AS NVARCHAR(MAX),
    @query  AS NVARCHAR(MAX);

SET @cols = (SELECT STRING_AGG(category,',') FROM (SELECT DISTINCT category FROM temp WHERE category IS NOT NULL)t);

set @query = 'SELECT date, ' + @cols + ' from 
            (
                select date
                    , amount
                    , category
                from temp
           ) x
            pivot 
            (
                 max(amount)
                for category in (' + @cols + ')
            ) p ';

execute(@query);

drop table temp;

How to produce an csv output file from stored procedure in SQL Server

You can do this using OPENROWSET as suggested in this answer. Reposting Slogmeister Extrarodinare answer:

Use T-SQL

INSERT INTO OPENROWSET ('Microsoft.ACE.OLEDB.12.0','Text;Database=D:\;HDR=YES;FMT=Delimited','SELECT * FROM [FileName.csv]')
SELECT Field1, Field2, Field3 FROM DatabaseName

But, there are couple of caveats:

  1. You need to have the Microsoft.ACE.OLEDB.12.0 provider available. The Jet 4.0 provider will work, too, but it's ancient, so I used this one instead.

  2. The .CSV file will have to exist already. If you're using headers (HDR=YES), make sure the first line of the .CSV file is a delimited list of all the fields.

An exception of type 'System.Data.SqlClient.SqlException' occurred in System.Data.dll

An unhandled exception of type 'System.Data.SqlClient.SqlException' occurred in System.Data.dll

    private const string strconneciton = "Data Source=.;Initial Catalog=Employees;Integrated Security=True";
    SqlConnection con = new SqlConnection(strconneciton);

    private void button1_Click(object sender, EventArgs e)
    {

        con.Open();
        SqlCommand cmd = new SqlCommand("insert into EmployeeData (Name,S/O,Address,Phone,CellNo,CNICNO,LicenseNo,LicenseDistrict,LicenseValidPhoto,ReferenceName,ReferenceContactNo) values ( '" + textName.Text + "','" + textSO.Text + "','" + textAddress.Text + "','" + textPhone.Text + "','" + textCell.Text + "','" + textCNIC.Text + "','" + textLicenseNo.Text + "','" + textLicenseDistrict.Text + "','" + textLicensePhoto.Text + "','" + textReferenceName.Text + "','" + textContact.Text + "' )", con);
        cmd.ExecuteNonQuery();
        con.Close();
    }

How to update two tables in one statement in SQL Server 2005?

You can write update statement for one table and then a trigger on first table update, which update second table

Convert Rows to columns using 'Pivot' in SQL Server

Just give you some idea how other databases solve this problem. DolphinDB also has built-in support for pivoting and the sql looks much more intuitive and neat. It is as simple as specifying the key column (Store), pivoting column (Week), and the calculated metric (sum(xCount)).

//prepare a 10-million-row table
n=10000000
t=table(rand(100, n) + 1 as Store, rand(54, n) + 1 as Week, rand(100, n) + 1 as xCount)

//use pivot clause to generate a pivoted table pivot_t
pivot_t = select sum(xCount) from t pivot by Store, Week

DolphinDB is a columnar high performance database. The calculation in the demo costs as low as 546 ms on a dell xps laptop (i7 cpu). To get more details, please refer to online DolphinDB manual https://www.dolphindb.com/help/index.html?pivotby.html

Return value in SQL Server stored procedure

Try to call your proc in this way:

DECLARE @UserIDout int

EXEC YOURPROC @EmailAddress = 'sdfds', @NickName = 'sdfdsfs', ..., @UserId = @UserIDout OUTPUT

SELECT @UserIDout 

Removing the remembered login and password list in SQL Server Management Studio

This works for SQL Server Management Studio v18.0

The file "SqlStudio.bin" doesn't seem to exist any longer. Instead my settings are all stored in this file:

C:\Users\*********\AppData\Roaming\Microsoft\SQL Server Management Studio\18.0\UserSettings.xml

  • Open it in any Texteditor like Notepad++
  • ctrl+f for the username to be removed
  • then delete the entire <Element>.......</Element> block that surrounds it.

SQL Server IF NOT EXISTS Usage?

Have you verified that there is in fact a row where Staff_Id = @PersonID? What you've posted works fine in a test script, assuming the row exists. If you comment out the insert statement, then the error is raised.

set nocount on

create table Timesheet_Hours (Staff_Id int, BookedHours int, Posted_Flag bit)

insert into Timesheet_Hours (Staff_Id, BookedHours, Posted_Flag) values (1, 5.5, 0)

declare @PersonID int
set @PersonID = 1

IF EXISTS    
    (
    SELECT 1    
    FROM Timesheet_Hours    
    WHERE Posted_Flag = 1    
        AND Staff_Id = @PersonID    
    )    
    BEGIN
        RAISERROR('Timesheets have already been posted!', 16, 1)
        ROLLBACK TRAN
    END
ELSE
    IF NOT EXISTS
        (
        SELECT 1
        FROM Timesheet_Hours
        WHERE Staff_Id = @PersonID
        )
        BEGIN
            RAISERROR('Default list has not been loaded!', 16, 1)
            ROLLBACK TRAN
        END
    ELSE
        print 'No problems here'

drop table Timesheet_Hours

Connect to SQL Server Database from PowerShell

Change Integrated security to false in the connection string.

You can check/verify this by opening up the SQL management studio with the username/password you have and see if you can connect/open the database from there. NOTE! Could be a firewall issue as well.

How do I compare 2 rows from the same table (SQL Server)?

SELECT COUNT(*) FROM (SELECT * FROM tbl WHERE id=1 UNION SELECT * FROM tbl WHERE id=2) a

If you got two rows, they different, if one - the same.

Referring to a Column Alias in a WHERE Clause

If you want to use the alias in your WHERE clause, you need to wrap it in a sub select, or CTE:

WITH LogDateDiff AS
(
   SELECT logcount, logUserID, maxlogtm
      , DATEDIFF(day, maxlogtm, GETDATE()) AS daysdiff
   FROM statslogsummary
)
SELECT logCount, logUserId, maxlogtm, daysdiff
FROM LogDateDiff
WHERE daysdiff > 120

SQL- Ignore case while searching for a string

See this similar question and answer to searching with case insensitivity - SQL server ignore case in a where expression

Try using something like:

SELECT DISTINCT COL_NAME 
FROM myTable 
WHERE COL_NAME COLLATE SQL_Latin1_General_CP1_CI_AS LIKE '%priceorder%'

Conversion failed when converting the varchar value to data type int in sql

The line

SELECT  @Prefix + LEN(CAST(@maxCode AS VARCHAR(10))+1) + CAST(@maxCode AS VARCHAR(100))

is wrong.

@Prefix is 'J' and LEN(...anything...) is an int, hence the type mismatch.


It seems to me, you actually want to do,

SELECT
        @maxCode = MAX(
            CAST(SUBSTRING(
                Voucher_No,
                @startFrom + 1,
                LEN(Voucher_No) - (@startFrom + 1)) AS INT)
    FROM
        dbo.Journal_Entry;

SELECT  @Prefix + CAST(@maxCode AS VARCHAR(10));

but, I couldn't say. If you illustrated before and after data, it would help.

Delete the 'first' record from a table in SQL Server, without a WHERE condition

WITH  q AS
        (
        SELECT TOP 1 *
        FROM    mytable
        /* You may want to add ORDER BY here */
        )
DELETE
FROM    q

Note that

DELETE TOP (1)
FROM   mytable

will also work, but, as stated in the documentation:

The rows referenced in the TOP expression used with INSERT, UPDATE, or DELETE are not arranged in any order.

Therefore, it's better to use WITH and an ORDER BY clause, which will let you specify more exactly which row you consider to be the first.

Visual Studio 2017 does not have Business Intelligence Integration Services/Projects

Integration Services project templates are now available in the latest release of SSDT for Visual Studio 2017.

Note: if you have recently installed SSDT for Visual Studio 2017. You need to remove the Reporting Services and Analysis Services installations before you proceed with installing SSDT.

SQL Update with row_number()

I did this for my situation and worked

WITH myUpdate (id, myRowNumber )
AS
( 
    SELECT id, ROW_NUMBER() over (order by ID) As myRowNumber
    FROM AspNetUsers
    WHERE  UserType='Customer' 
 )

update AspNetUsers set EmployeeCode = FORMAT(myRowNumber,'00000#') 
FROM myUpdate
    left join AspNetUsers u on u.Id=myUpdate.id

Inner Joining three tables

dbo.tableA AS A INNER JOIN dbo.TableB AS B
ON A.common = B.common INNER JOIN TableC C
ON B.common = C.common

CASE IN statement with multiple values

If you have more numbers or if you intend to add new test numbers for CASE then you can use a more flexible approach:

DECLARE @Numbers TABLE
(
    Number VARCHAR(50) PRIMARY KEY
    ,Class TINYINT NOT NULL
);
INSERT @Numbers
VALUES ('1121231',1);
INSERT @Numbers
VALUES ('31242323',1);
INSERT @Numbers
VALUES ('234523',2);
INSERT @Numbers
VALUES ('2342423',2);

SELECT c.*, n.Class
FROM   tblClient c  
LEFT OUTER JOIN   @Numbers n ON c.Number = n.Number;

Also, instead of table variable you can use a regular table.

Format number as percent in MS SQL Server

SELECT cast( cast(round(37.0/38.0,2) AS DECIMAL(18,2)) as varchar(100)) + ' %'

RESULT:  0.97 %

What do Clustered and Non clustered index actually mean?

Clustered Index: Primary Key constraint creates clustered Index automatically if no clustered Index already exists on the table. Actual data of clustered index can be stored at leaf level of Index.

Non Clustered Index: Actual data of non clustered index is not directly found at leaf node, instead it has to take an additional step to find because it has only values of row locators pointing towards actual data. Non clustered Index can't be sorted as clustered index. There can be multiple non clustered indexes per table, actually it depends on the sql server version we are using. Basically Sql server 2005 allows 249 Non Clustered Indexes and for above versions like 2008, 2016 it allows 999 Non Clustered Indexes per table.

How to get calendar Quarter from a date in TSQL

Assuming field data type INT and field name "mydate". In the OP question, the INT date values when converted to string are ISO date literals.

select DatePart(QUARTER, cast(cast(mydate as char(8)) as date))

You can use datetime if using older server version.

How to read the last row with SQL Server

If you're using MS SQL, you can try:

SELECT TOP 1 * FROM table_Name ORDER BY unique_column DESC 

Return sql rows where field contains ONLY non-alphanumeric characters

SQL Server doesn't have regular expressions. It uses the LIKE pattern matching syntax which isn't the same.

As it happens, you are close. Just need leading+trailing wildcards and move the NOT

 WHERE whatever NOT LIKE '%[a-z0-9]%'

How to Identify port number of SQL server

You can also use this query

USE MASTER GO xp_readerrorlog 0, 1, N'Server is listening on' GO

Source : sqlauthority blog

Violation of PRIMARY KEY constraint. Cannot insert duplicate key in object

Not OP's answer but as this was the first question that popped up for me in google, Id also like to add that users searching for this might need to reseed their table, which was the case for me

DBCC CHECKIDENT(tablename)

Declare Variable for a Query String

It's possible, but it requires using dynamic SQL.
I recommend reading The curse and blessings of dynamic SQL before continuing...

DECLARE @theDate varchar(60)
SET @theDate = '''2010-01-01'' AND ''2010-08-31 23:59:59'''

DECLARE @SQL VARCHAR(MAX)  
SET @SQL = 'SELECT AdministratorCode, 
                   SUM(Total) as theTotal, 
                   SUM(WOD.Quantity) as theQty, 
                   AVG(Total) as avgTotal, 
                  (SELECT SUM(tblWOD.Amount)
                     FROM tblWOD
                     JOIN tblWO on tblWOD.OrderID = tblWO.ID
                    WHERE tblWO.Approved = ''1''
                      AND tblWO.AdministratorCode = tblWO.AdministratorCode
                      AND tblWO.OrderDate BETWEEN '+ @theDate +')'

EXEC(@SQL)

Dynamic SQL is just a SQL statement, composed as a string before being executed. So the usual string concatenation occurs. Dynamic SQL is required whenever you want to do something in SQL syntax that isn't allowed, like:

  • a single parameter to represent comma separated list of values for an IN clause
  • a variable to represent both value and SQL syntax (IE: the example you provided)

EXEC sp_executesql allows you to use bind/preparedstatement parameters so you don't have to concern yourself with escaping single quotes/etc for SQL injection attacks.

How to kill/stop a long SQL query immediately?

If you cancel and see that run

 sp_who2 'active'

(Activity Monitor won't be available on old sql server 2000 FYI )

Spot the SPID you wish to kill e.g. 81

Kill 81

Run the sp_who2 'active' again and you will probably notice it is sleeping ... rolling back

To get the STATUS run again the KILL

Kill 81 

Then you will get a message like this

 SPID 81: transaction rollback in progress. Estimated rollback completion: 63%. Estimated time remaining: 992 seconds.

Create a view with ORDER BY clause

As one of the comments in this posting suggests using stored procedures to return the data... I think that is the best answer. In my case what I did is wrote a View to encapsulate the query logic and joins, then I wrote a Stored Proc to return the data sorted and the proc also includes other enhancement features such as parameters for filtering the data.

Now you have to option to query the view, which allows you to manipulate the data further. Or you have the option to execute the stored proc, which is quicker and more precise output.

STORED PROC Execution to query data

exec [olap].[uspUsageStatsLogSessionsRollup]

VIEW Definition

USE [DBA]
GO

/****** Object:  View [olap].[vwUsageStatsLogSessionsRollup]    Script Date: 2/19/2019 10:10:06 AM ******/
SET ANSI_NULLS ON
GO

SET QUOTED_IDENTIFIER ON
GO


--USE DBA
-- select * from olap.UsageStatsLog_GCOP039 where CubeCommand='[ORDER_HISTORY]'
;

ALTER VIEW [olap].[vwUsageStatsLogSessionsRollup] as
(
    SELECT --*
        t1.UsageStatsLogDate
        , COALESCE(CAST(t1.UsageStatsLogDate AS nvarchar(100)), 'TOTAL- DATES:') AS UsageStatsLogDate_Totals
        , t1.ADUserNameDisplayNEW
        , COALESCE(t1.ADUserNameDisplayNEW, 'TOTAL- USERS:') AS ADUserNameDisplay_Totals
        , t1.CubeCommandNEW
        , COALESCE(t1.CubeCommandNEW, 'TOTAL- CUBES:') AS CubeCommand_Totals
        , t1.SessionsCount
        , t1.UsersCount
        , t1.CubesCount
    FROM
    (
        select 
            CAST(olapUSL.UsageStatsLogTime as date) as UsageStatsLogDate
            , olapUSL.ADUserNameDisplayNEW
            , olapUSL.CubeCommandNEW
            , count(*) SessionsCount
            , count(distinct olapUSL.ADUserNameDisplayNEW) UsersCount
            , count(distinct olapUSL.CubeCommandNEW) CubesCount
        from 
            olap.vwUsageStatsLog olapUSL
        where CubeCommandNEW != '[]'
        GROUP BY CUBE(CAST(olapUSL.UsageStatsLogTime as date), olapUSL.ADUserNameDisplayNEW, olapUSL.CubeCommandNEW )
            ----GROUP BY 
            ------GROUP BY GROUPING SETS
            --------GROUP BY ROLLUP
    ) t1

    --ORDER BY
    --  t1.UsageStatsLogDate DESC
    --  , t1.ADUserNameDisplayNEW
    --  , t1.CubeCommandNEW
)
;


GO

STORED PROC Definition

USE [DBA]
GO

/****** Object:  StoredProcedure [olap].[uspUsageStatsLogSessionsRollup]    Script Date: 2/19/2019 9:39:31 AM ******/
SET ANSI_NULLS ON
GO

SET QUOTED_IDENTIFIER ON
GO


-- =============================================
-- Author:      BRIAN LOFTON
-- Create date: 2/19/2019
-- Description: This proceedured returns data from a view with sorted results and an optional date range filter.
-- =============================================
ALTER PROCEDURE [olap].[uspUsageStatsLogSessionsRollup]
    -- Add the parameters for the stored procedure here
    @paramStartDate date = NULL,
    @paramEndDate date = NULL,
    @paramDateTotalExcluded as int = 0,
    @paramUserTotalExcluded as int = 0,
    @paramCubeTotalExcluded as int = 0
AS

BEGIN
    -- SET NOCOUNT ON added to prevent extra result sets from interfering with SELECT statements.
    SET NOCOUNT ON;

    DECLARE @varStartDate as date 
        = CASE  
            WHEN @paramStartDate IS NULL THEN '1900-01-01' 
            ELSE @paramStartDate 
        END
    DECLARE @varEndDate as date 
        = CASE  
            WHEN @paramEndDate IS NULL THEN '2100-01-01' 
            ELSE @paramStartDate 
        END

    -- Return Data from this statement
    SELECT 
        t1.UsageStatsLogDate_Totals
        , t1.ADUserNameDisplay_Totals
        , t1.CubeCommand_Totals
        , t1.SessionsCount
        , t1.UsersCount
        , t1.CubesCount
        -- Fields with NULL in the totals
            --  , t1.CubeCommandNEW
            --  , t1.ADUserNameDisplayNEW
            --  , t1.UsageStatsLogDate
    FROM 
        olap.vwUsageStatsLogSessionsRollup t1
    WHERE

        (
            --t1.UsageStatsLogDate BETWEEN @varStartDate AND @varEndDate
            t1.UsageStatsLogDate BETWEEN '1900-01-01' AND '2100-01-01'
            OR t1.UsageStatsLogDate IS NULL
        )
        AND
        (
            @paramDateTotalExcluded=0
            OR (@paramDateTotalExcluded=1 AND UsageStatsLogDate_Totals NOT LIKE '%TOTAL-%')
        )
        AND
        (
            @paramDateTotalExcluded=0
            OR (@paramUserTotalExcluded=1 AND ADUserNameDisplay_Totals NOT LIKE '%TOTAL-%')
        )
        AND
        (
            @paramCubeTotalExcluded=0
            OR (@paramCubeTotalExcluded=1 AND CubeCommand_Totals NOT LIKE '%TOTAL-%')
        )
    ORDER BY
            t1.UsageStatsLogDate DESC
            , t1.ADUserNameDisplayNEW
            , t1.CubeCommandNEW

END


GO

How to use PHP to connect to sql server

Try this code

$serverName = "serverName\sqlexpress"; //serverName\instanceName
$connectionInfo = array( "Database"=>"dbName", "UID"=>"userName", "PWD"=>"password");
$conn = sqlsrv_connect( $serverName, $connectionInfo);

How to BULK INSERT a file into a *temporary* table where the filename is a variable?

Sorry to dig up an old question but in case someone stumbles onto this thread and wants a quicker solution.

Bulk inserting a unknown width file with \n row terminators into a temp table that is created outside of the EXEC statement.

DECLARE     @SQL VARCHAR(8000)

IF OBJECT_ID('TempDB..#BulkInsert') IS NOT NULL
BEGIN
    DROP TABLE #BulkInsert
END

CREATE TABLE #BulkInsert
(
    Line    VARCHAR(MAX)
)

SET @SQL = 'BULK INSERT #BulkInser FROM ''##FILEPATH##'' WITH (ROWTERMINATOR = ''\n'')'
EXEC (@SQL)

SELECT * FROM #BulkInsert

Further support that dynamic SQL within an EXEC statement has access to temp tables outside of the EXEC statement. http://sqlfiddle.com/#!3/d41d8/19343

DECLARE     @SQL VARCHAR(8000)

IF OBJECT_ID('TempDB..#BulkInsert') IS NOT NULL
BEGIN
    DROP TABLE #BulkInsert
END

CREATE TABLE #BulkInsert
(
    Line    VARCHAR(MAX)
)
INSERT INTO #BulkInsert
(
    Line
)
SELECT 1
UNION SELECT 2
UNION SELECT 3

SET @SQL = 'SELECT * FROM #BulkInsert'
EXEC (@SQL)

Further support, written for MSSQL2000 http://technet.microsoft.com/en-us/library/aa175921(v=sql.80).aspx

Example at the bottom of the link

DECLARE @cmd VARCHAR(1000), @ExecError INT
CREATE TABLE #ErrFile (ExecError INT)
SET @cmd = 'EXEC GetTableCount ' + 
'''pubs.dbo.authors''' + 
'INSERT #ErrFile VALUES(@@ERROR)'
EXEC(@cmd)
SET @ExecError = (SELECT * FROM #ErrFile)
SELECT @ExecError AS '@@ERROR'

Count all duplicates of each value

This is quite simple.

Assuming the data is stored in a column called A in a table called T, you can use

select A, count(A) from T group by A

Get the week start date and week end date from week number

You can find the day of week and do a date add on days to get the start and end dates..

DATEADD(dd, -(DATEPART(dw, WeddingDate)-1), WeddingDate) [WeekStart]

DATEADD(dd, 7-(DATEPART(dw, WeddingDate)), WeddingDate) [WeekEnd]

You probably also want to look at stripping off the time from the date as well though.

Search text in stored procedure in SQL Server

Every so often I use this script to figure out which procs to modify, or to figure out what uses a column of a table, or that table at all to remove some old junk. It checks each database on the instance it is ran on by the wonderfully supplied sp_msforeachdb.

if object_id('tempdb..##nothing') is not null
    drop table ##nothing

CREATE TABLE ##nothing
(
    DatabaseName varchar(30),
    SchemaName varchar(30),
    ObjectName varchar(100),
    ObjectType varchar(50)
)

EXEC master.sys.sp_msforeachdb 
'USE ?
insert into ##nothing
SELECT 
db_name() AS [Database],
[Scehma]=schema_name(o.schema_id), 
o.Name, 
o.type 
FROM sys.sql_modules m
INNER JOIN sys.objects o
    ON o.object_id = m.object_id
WHERE 
    m.definition like ''%SOME_TEXT%'''  
--edit this text

SELECT * FROM ##nothing n
order by OBJECTname 

Storing JSON in database vs. having a new column for each key

You are trying to fit a non-relational model into a relational database, I think you would be better served using a NoSQL database such as MongoDB. There is no predefined schema which fits in with your requirement of having no limitation to the number of fields (see the typical MongoDB collection example). Check out the MongoDB documentation to get an idea of how you'd query your documents, e.g.

db.mycollection.find(
    {
      name: 'sann'
    }
)

What is a 'multi-part identifier' and why can't it be bound?

Binding = your textual representation of a specific column gets mapped to a physical column in some table, in some database, on some server.

Multipart identifier could be: MyDatabase.dbo.MyTable. If you get any of these identifiers wrong, then you have a multipart identifier that cannot be mapped.

The best way to avoid it is to write the query right the first time, or use a plugin for management studio that provides intellisense and thus help you out by avoiding typos.

Regular expressions inside SQL Server

stored value in DB is: 5XXXXXX [where x can be any digit]

You don't mention data types - if numeric, you'll likely have to use CAST/CONVERT to change the data type to [n]varchar.

Use:

WHERE CHARINDEX(column, '5') = 1
  AND CHARINDEX(column, '.') = 0 --to stop decimals if needed
  AND ISNUMERIC(column) = 1

References:

i have also different cases like XXXX7XX for example, so it has to be generic.

Use:

WHERE PATINDEX('%7%', column) = 5
  AND CHARINDEX(column, '.') = 0 --to stop decimals if needed
  AND ISNUMERIC(column) = 1

References:

Regex Support

SQL Server 2000+ supports regex, but the catch is you have to create the UDF function in CLR before you have the ability. There are numerous articles providing example code if you google them. Once you have that in place, you can use:

  • 5\d{6} for your first example
  • \d{4}7\d{2} for your second example

For more info on regular expressions, I highly recommend this website.

How to undo a SQL Server UPDATE query?

A non-committed transaction can be reverted by issuing the command ROLLBACK

But if you are running in auto-commit mode there is nothing you can do....

Query to list number of records in each table in a database

USE DatabaseName
CREATE TABLE #counts
(
    table_name varchar(255),
    row_count int
)

EXEC sp_MSForEachTable @command1='INSERT #counts (table_name, row_count) SELECT ''?'', COUNT(*) FROM ?'
SELECT table_name, row_count FROM #counts ORDER BY table_name, row_count DESC
DROP TABLE #counts

How to find sum of multiple columns in a table in SQL Server 2005?

use a trigges it will work:-

->CREATE TRIGGER trigger_name BEFORE INSERT ON table_name

FOR EACH ROW SET NEW.column_name3 = NEW.column_name1 + NEW.column_name2;

this will only work only when you will insert a row in table not when you will be updating your table for such a pupose create another trigger of different name and use UPDATE on the place of INSERT in the above syntax

How can I group by date time column without taking time into consideration

CAST datetime field to date

select  CAST(datetime_field as DATE), count(*) as count from table group by CAST(datetime_field as DATE);

Check if starting characters of a string are alphabetical in T-SQL

select * from my_table where my_field Like '[a-z][a-z]%'

String or binary data would be truncated. The statement has been terminated

SQL Server 2016 SP2 CU6 and SQL Server 2017 CU12 introduced trace flag 460 in order to return the details of truncation warnings. You can enable it at the query level or at the server level.

Query level

INSERT INTO dbo.TEST (ColumnTest)
VALUES (‘Test truncation warnings’)
OPTION (QUERYTRACEON 460);
GO

Server Level

DBCC TRACEON(460, -1);
GO

From SQL Server 2019 you can enable it at database level:

ALTER DATABASE SCOPED CONFIGURATION 
SET VERBOSE_TRUNCATION_WARNINGS = ON;

The old output message is:

Msg 8152, Level 16, State 30, Line 13
String or binary data would be truncated.
The statement has been terminated.

The new output message is:

Msg 2628, Level 16, State 1, Line 30
String or binary data would be truncated in table 'DbTest.dbo.TEST', column 'ColumnTest'. Truncated value: ‘Test truncation warnings‘'.

In a future SQL Server 2019 release, message 2628 will replace message 8152 by default.

When increasing the size of VARCHAR column on a large table could there be any problems?

Changing to Varchar(1200) from Varchar(200) should cause you no issue as it is only a metadata change and as SQL server 2008 truncates excesive blank spaces you should see no performance differences either so in short there should be no issues with making the change.

How do I find the data directory for a SQL Server instance?

Keeping it simple:

use master
select DB.name, F.physical_name from sys.databases DB join sys.master_files F on DB.database_id=F.database_id

this will return all databases with associated files

Like Operator in Entity Framework?

I don't know anything about EF really, but in LINQ to SQL you usually express a LIKE clause using String.Contains:

where entity.Name.Contains("xyz")

translates to

WHERE Name LIKE '%xyz%'

(Use StartsWith and EndsWith for other behaviour.)

I'm not entirely sure whether that's helpful, because I don't understand what you mean when you say you're trying to implement LIKE. If I've misunderstood completely, let me know and I'll delete this answer :)

How to remove foreign key constraint in sql server?

If you don't know foreign key constraint name then try this to find it.

sp_help 'TableName'   

additionally for different schema

sp_help 'schemaName.TableName'   

then

ALTER TABLE <TABLE_NAME> DROP CONSTRAINT <FOREIGN_KEY_NAME>

how to set the query timeout from SQL connection string

Only from code:

_x000D_
_x000D_
namespace xxx.DsXxxTableAdapters {_x000D_
    partial class ZzzTableAdapter_x000D_
    {_x000D_
        public void SetTimeout(int timeout)_x000D_
        {_x000D_
            if (this.Adapter.DeleteCommand != null) { this.Adapter.DeleteCommand.CommandTimeout = timeout; }_x000D_
            if (this.Adapter.InsertCommand != null) { this.Adapter.InsertCommand.CommandTimeout = timeout; }_x000D_
            if (this.Adapter.UpdateCommand != null) { this.Adapter.UpdateCommand.CommandTimeout = timeout; }_x000D_
            if (this._commandCollection == null) { this.InitCommandCollection(); }_x000D_
            if (this._commandCollection != null)_x000D_
            {_x000D_
                foreach (System.Data.SqlClient.SqlCommand item in this._commandCollection)_x000D_
                {_x000D_
                    if (item != null)_x000D_
                    { item.CommandTimeout = timeout; }_x000D_
                }_x000D_
            }_x000D_
        }_x000D_
    }_x000D_
 _x000D_
    //...._x000D_
 _x000D_
 }
_x000D_
_x000D_
_x000D_

I want to use CASE statement to update some records in sql server 2005

This is also an alternate use of case-when...

UPDATE [dbo].[JobTemplates]
SET [CycleId] = 
    CASE [Id]
        WHEN 1376 THEN 44   --ACE1 FX1
        WHEN 1385 THEN 44   --ACE1 FX2
        WHEN 1574 THEN 43   --ACE1 ELEM1
        WHEN 1576 THEN 43   --ACE1 ELEM2
        WHEN 1581 THEN 41   --ACE1 FS1
        WHEN 1585 THEN 42   --ACE1 HS1
        WHEN 1588 THEN 43   --ACE1 RS1
        WHEN 1589 THEN 44   --ACE1 RM1
        WHEN 1590 THEN 43   --ACE1 ELEM3
        WHEN 1591 THEN 43   --ACE1 ELEM4
        WHEN 1595 THEN 44   --ACE1 SSTn     
        ELSE 0  
     END
WHERE
    [Id] IN (1376,1385,1574,1576,1581,1585,1588,1589,1590,1591,1595)

I like the use of the temporary tables in cases where duplicate values are not permitted and your update may create them. For example:

SELECT
     [Id]
    ,[QueueId]
    ,[BaseDimensionId]
    ,[ElastomerTypeId]
    ,CASE [CycleId]
        WHEN  29 THEN 44
        WHEN  30 THEN 43
        WHEN  31 THEN 43
        WHEN 101 THEN 41
        WHEN 102 THEN 43
        WHEN 116 THEN 42
        WHEN 120 THEN 44
        WHEN 127 THEN 44
        WHEN 129 THEN 44
        ELSE    0
     END                AS [CycleId]
INTO
    ##ACE1_PQPANominals_1
FROM 
    [dbo].[ProductionQueueProcessAutoclaveNominals]
WHERE
    [QueueId] = 3
ORDER BY 
    [BaseDimensionId], [ElastomerTypeId], [Id];
---- (403 row(s) affected)

UPDATE [dbo].[ProductionQueueProcessAutoclaveNominals]
SET 
    [CycleId] = X.[CycleId]
FROM
    [dbo].[ProductionQueueProcessAutoclaveNominals]
INNER JOIN
(
    SELECT  
        MIN([Id]) AS [Id],[QueueId],[BaseDimensionId],[ElastomerTypeId],[CycleId] 
    FROM 
        ##ACE1_PQPANominals_1
    GROUP BY    
        [QueueId],[BaseDimensionId],[ElastomerTypeId],[CycleId] 
) AS X
ON
    [dbo].[ProductionQueueProcessAutoclaveNominals].[Id] = X.[Id];
----(375 row(s) affected)

How to give a user only select permission on a database

You could add the user to the Database Level Role db_datareader.

Members of the db_datareader fixed database role can run a SELECT statement against any table or view in the database.

See Books Online for reference:

http://msdn.microsoft.com/en-us/library/ms189121%28SQL.90%29.aspx

You can add a database user to a database role using the following query:

EXEC sp_addrolemember N'db_datareader', N'userName'

How to convert DateTime to VarChar

Try:

select replace(convert(varchar, getdate(), 111),'/','-');

More on ms sql tips

How do I return the SQL data types from my query?

This easy query return a data type bit. You can use this thecnic for other data types:

select CAST(0 AS BIT) AS OK

How to trim a string in SQL Server before 2017?

SELECT LTRIM(RTRIM(Names)) AS Names FROM Customer

How can I escape square brackets in a LIKE clause?

I needed to exclude names that started with an underscore from a query, so I ended up with this:

WHERE b.[name] not like '\_%' escape '\'  -- use \ as the escape character

How to find third or n?? maximum salary from salary table?

select *  from emp x where &no=(select count(*) from emp y where y.sal>=x.sal);

this will give take input from user and then will tell the nth maximum number.I have taken example of emp table in oracle and to display nth maximum salaried employee info

Output Enter value for no: 5

 EMPNO ENAME      JOB       MGR        HIREDATE  SAL        COMM       DEPTNO
 ----- ---------- --------- ---------- --------- ---------- ---------- ----------
 7698  BLAKE      MANAGER   7839       01-MAY-81 3000                  30
 7788  SCOTT      ANALYST   7566       19-APR-87 3000                  20
 7902  FORD       ANALYST   7566       03-DEC-81 3000                  20

Enter value for no: 14

 EMPNO ENAME      JOB       MGR        HIREDATE  SAL        COMM       DEPTNO
 ----- ---------- --------- ---------- --------- ---------- ---------- ----------
 7369  SMITH      CLERK     7902       17-DEC-80 800                   20

GROUP BY to combine/concat a column

A good question. Should tell you it took some time to crack this one. Here is my result.

DECLARE @TABLE TABLE
(  
ID INT,  
USERS VARCHAR(10),  
ACTIVITY VARCHAR(10),  
PAGEURL VARCHAR(10)  
)

INSERT INTO @TABLE  
VALUES  (1, 'Me', 'act1', 'ab'),
        (2, 'Me', 'act1', 'cd'),
        (3, 'You', 'act2', 'xy'),
        (4, 'You', 'act2', 'st')


SELECT T1.USERS, T1.ACTIVITY,   
        STUFF(  
        (  
        SELECT ',' + T2.PAGEURL  
        FROM @TABLE T2  
        WHERE T1.USERS = T2.USERS  
        FOR XML PATH ('')  
        ),1,1,'')  
FROM @TABLE T1  
GROUP BY T1.USERS, T1.ACTIVITY

What is the command to truncate a SQL Server log file?

For SQL Server 2008, the command is:

ALTER DATABASE ExampleDB SET RECOVERY SIMPLE
DBCC SHRINKFILE('ExampleDB_log', 0, TRUNCATEONLY)
ALTER DATABASE ExampleDB SET RECOVERY FULL

This reduced my 14GB log file down to 1MB.

What is the connection string for localdb for version 11

In Sql Server 2008 R2 database files you can connect with

Server=np:\\.\pipe\YourInstance\tsql\query;InitialCatalog=yourDataBase;Trusted_Connection=True;

only, but in sql Server 2012 you can use this:

Server=(localdb)\v11.0;Integrated Security=true;Database=DB1;

and it depended on your .mdf .ldf version.

for finding programmicaly i use this Method that explained in this post

TABLOCK vs TABLOCKX

This is more of an example where TABLOCK did not work for me and TABLOCKX did.

I have 2 sessions, that both use the default (READ COMMITTED) isolation level:

Session 1 is an explicit transaction that will copy data from a linked server to a set of tables in a database, and takes a few seconds to run. [Example, it deletes Questions] Session 2 is an insert statement, that simply inserts rows into a table that Session 1 doesn't make changes to. [Example, it inserts Answers].

(In practice there are multiple sessions inserting multiple records into the table, simultaneously, while Session 1 is running its transaction).

Session 1 has to query the table Session 2 inserts into because it can't delete records that depend on entries that were added by Session 2. [Example: Delete questions that have not been answered].

So, while Session 1 is executing and Session 2 tries to insert, Session 2 loses in a deadlock every time.

So, a delete statement in Session 1 might look something like this: DELETE tblA FROM tblQ LEFT JOIN tblX on ... LEFT JOIN tblA a ON tblQ.Qid = tblA.Qid WHERE ... a.QId IS NULL and ...

The deadlock seems to be caused from contention between querying tblA while Session 2, [3, 4, 5, ..., n] try to insert into tblA.

In my case I could change the isolation level of Session 1's transaction to be SERIALIZABLE. When I did this: The transaction manager has disabled its support for remote/network transactions.

So, I could follow instructions in the accepted answer here to get around it: The transaction manager has disabled its support for remote/network transactions

But a) I wasn't comfortable with changing the isolation level to SERIALIZABLE in the first place- supposedly it degrades performance and may have other consequences I haven't considered, b) didn't understand why doing this suddenly caused the transaction to have a problem working across linked servers, and c) don't know what possible holes I might be opening up by enabling network access.

There seemed to be just 6 queries within a very large transaction that are causing the trouble.

So, I read about TABLOCK and TabLOCKX.

I wasn't crystal clear on the differences, and didn't know if either would work. But it seemed like it would. First I tried TABLOCK and it didn't seem to make any difference. The competing sessions generated the same deadlocks. Then I tried TABLOCKX, and no more deadlocks.

So, in six places, all I needed to do was add a WITH (TABLOCKX).

So, a delete statement in Session 1 might look something like this: DELETE tblA FROM tblQ q LEFT JOIN tblX x on ... LEFT JOIN tblA a WITH (TABLOCKX) ON tblQ.Qid = tblA.Qid WHERE ... a.QId IS NULL and ...

How to convert SQL Server's timestamp column to datetime format

Why not try FROM_UNIXTIME(unix_timestamp, format)?

How to declare a variable in SQL Server and use it in the same Stored Procedure

CREATE PROCEDURE AddBrand
@BrandName nvarchar(50) = null,
@CategoryID int = null
AS    
BEGIN

DECLARE @BrandID int = null
SELECT @BrandID = BrandID FROM tblBrand 
WHERE BrandName = @BrandName

INSERT INTO tblBrandinCategory (CategoryID, BrandID) 
       VALUES (@CategoryID, @BrandID)

END

EXEC AddBrand @BrandName = 'BMW', @CategoryId = 1

Remote Procedure call failed with sql server 2008 R2

Open Control Panel Administrative Tools Services Select Extended services tab

Find SQL Server(MSSQLSERVER) & SQL Server(SQLEXPRESS) Stop these services and Start again (from the start & stop button displayed above)

Done.

conversion of a varchar data type to a datetime data type resulted in an out-of-range value

Ambiguous date formats are interpreted according to the language of the login. This works

set dateformat mdy

select CAST('03/28/2011 18:03:40' AS DATETIME)

This doesn't

set dateformat dmy

select CAST('03/28/2011 18:03:40' AS DATETIME)

If you use parameterised queries with the correct datatype you avoid these issues. You can also use the unambiguous "unseparated" format yyyyMMdd hh:mm:ss

Is it necessary to use # for creating temp tables in SQL server?

Yes. You need to prefix the table name with "#" (hash) to create temporary tables.

If you do NOT need the table later, go ahead & create it. Temporary Tables are very much like normal tables. However, it gets created in tempdb. Also, it is only accessible via the current session i.e. For EG: if another user tries to access the temp table created by you, he'll not be able to do so.

"##" (double-hash creates "Global" temp table that can be accessed by other sessions as well.

Refer the below link for the Basics of Temporary Tables: http://www.codeproject.com/Articles/42553/Quick-Overview-Temporary-Tables-in-SQL-Server-2005

If the content of your table is less than 5000 rows & does NOT contain data types such as nvarchar(MAX), varbinary(MAX), consider using Table Variables.

They are the fastest as they are just like any other variables which are stored in the RAM. They are stored in tempdb as well, not in RAM.

DECLARE @ItemBack1 TABLE
(
 column1 int,
 column2 int,
 someInt int,
 someVarChar nvarchar(50)
);

INSERT INTO @ItemBack1
SELECT column1, 
       column2, 
       someInt, 
       someVarChar 
  FROM table2
 WHERE table2.ID = 7;

More Info on Table Variables: http://odetocode.com/articles/365.aspx

How to replace (null) values with 0 output in PIVOT

You cannot place the IsNull() until after the data is selected so you will place the IsNull() around the final value in the SELECT:

SELECT CLASS,
  IsNull([AZ], 0) as [AZ],
  IsNull([CA], 0) as [CA],
  IsNull([TX], 0) as [TX]
FROM #TEMP
PIVOT 
(
  SUM(DATA)
  FOR STATE IN ([AZ], [CA], [TX])
) AS PVT
ORDER BY CLASS

Update Query with INNER JOIN between tables in 2 different databases on 1 server

Following is the MySQL syntax:

UPDATE table1 
INNER JOIN table2 ON table1.field1 = table2.field2
SET table1.field3 = table2.field4 
WHERE ...... ;

http://geekswithblogs.net/faizanahmad/archive/2009/01/05/join-in-sql-update--statement.aspx

SQL Server stored procedure Nullable parameter

It looks like you're passing in Null for every argument except for PropertyValueID and DropDownOptionID, right? I don't think any of your IF statements will fire if only these two values are not-null. In short, I think you have a logic error.

Other than that, I would suggest two things...

First, instead of testing for NULL, use this kind syntax on your if statements (it's safer)...

    ELSE IF ISNULL(@UnitValue, 0) != 0 AND ISNULL(@UnitOfMeasureID, 0) = 0

Second, add a meaningful PRINT statement before each UPDATE. That way, when you run the sproc in MSSQL, you can look at the messages and see how far it's actually getting.

How to format a numeric column as phone number in SQL

This should do it:

UPDATE TheTable
SET PhoneNumber = SUBSTRING(PhoneNumber, 1, 3) + '-' + 
                  SUBSTRING(PhoneNumber, 4, 3) + '-' + 
                  SUBSTRING(PhoneNumber, 7, 4)

Incorporated Kane's suggestion, you can compute the phone number's formatting at runtime. One possible approach would be to use scalar functions for this purpose (works in SQL Server):

CREATE FUNCTION FormatPhoneNumber(@phoneNumber VARCHAR(10))
RETURNS VARCHAR(12)
BEGIN
    RETURN SUBSTRING(@phoneNumber, 1, 3) + '-' + 
           SUBSTRING(@phoneNumber, 4, 3) + '-' + 
           SUBSTRING(@phoneNumber, 7, 4)
END

Convert HashBytes to VarChar

With personal experience of using the following code within a Stored Procedure which Hashed a SP Variable I can confirm, although undocumented, this combination works 100% as per my example:

@var=SUBSTRING(master.dbo.fn_varbintohexstr(HashBytes('SHA2_512', @SPvar)), 3, 128)

Backup a single table with its data from a database in sql server 2008

There are many ways you can take back of table.

  1. BCP (BULK COPY PROGRAM)
  2. Generate Table Script with data
  3. Make a copy of table using SELECT INTO, example here
  4. SAVE Table Data Directly in a Flat file
  5. Export Data using SSIS to any destination

SQL Server Creating a temp table for this query

IF OBJECT_ID('tempdb..#MyTempTable') IS NOT NULL DROP TABLE #MyTempTable

CREATE TABLE #MyTempTable (SiteName varchar(50), BillingMonth varchar(10), Consumption float)

INSERT INTO #MyTempTable (SiteName, BillingMonth, Consumption)
SELECT  tblMEP_Sites.Name AS SiteName, convert(varchar(10),BillingMonth ,101)
AS BillingMonth, SUM(Consumption) AS Consumption
FROM tblMEP_Projects.......

Difference between datetime and timestamp in sqlserver?

Datetime is a datatype.

Timestamp is a method for row versioning. In fact, in sql server 2008 this column type was renamed (i.e. timestamp is deprecated) to rowversion. It basically means that every time a row is changed, this value is increased. This is done with a database counter which automatically increase for every inserted or updated row.

For more information:

http://www.sqlteam.com/article/timestamps-vs-datetime-data-types

http://msdn.microsoft.com/en-us/library/ms182776.aspx

T-SQL split string

here is a version that can split on a pattern using patindex, a simple adaptation of the post above. I had a case where I needed to split a string that contained multiple separator chars.


alter FUNCTION dbo.splitstring ( @stringToSplit VARCHAR(1000), @splitPattern varchar(10) )
RETURNS
 @returnList TABLE ([Name] [nvarchar] (500))
AS
BEGIN

 DECLARE @name NVARCHAR(255)
 DECLARE @pos INT

 WHILE PATINDEX(@splitPattern, @stringToSplit) > 0
 BEGIN
  SELECT @pos  = PATINDEX(@splitPattern, @stringToSplit)  
  SELECT @name = SUBSTRING(@stringToSplit, 1, @pos-1)

  INSERT INTO @returnList 
  SELECT @name

  SELECT @stringToSplit = SUBSTRING(@stringToSplit, @pos+1, LEN(@stringToSplit)-@pos)
 END

 INSERT INTO @returnList
 SELECT @stringToSplit

 RETURN
END
select * from dbo.splitstring('stringa/stringb/x,y,z','%[/,]%');

result looks like this

stringa stringb x y z

Select n random rows from SQL Server table

This is a combination of the initial seed idea and a checksum, which looks to me to give properly random results without the cost of NEWID():

SELECT TOP [number] 
FROM table_name
ORDER BY RAND(CHECKSUM(*) * RAND())

Connecting to SQL Server using windows authentication

Check out www.connectionstrings.com for a ton of samples of proper connection strings.

In your case, use this:

Server=localhost;Database=employeedetails;Integrated Security=SSPI

Update: obviously, the service account used to run ASP.NET web apps doesn't have access to SQL Server, and judging from that error message, you're probably using "anonymous authentication" on your web site.

So you either need to add this account IIS APPPOOL\ASP.NET V4.0 as a SQL Server login and give that login access to your database, or you need to switch to using "Windows authentication" on your ASP.NET web site so that the calling Windows account will be passed through to SQL Server and used as a login on SQL Server.

Select SQL Server database size

SELECT      sys.databases.name AS [Database Name],  
        CONVERT(VARCHAR,SUM(size)*8/1024)+' MB' AS [Size]  
     FROM        sys.databases   
     JOIN        sys.master_files  
     ON          sys.databases.database_id=sys.master_files.database_id  
     GROUP BY    sys.databases.name  
     ORDER BY    sys.databases.name  

Adding an identity to an existing column

I'm a java developer that happened to get on a team without a DBA and one where as a developer, I can't get DBA rights. I was tasked with moving an entire schema between two databases, so without having a DBA, I had to do it and do it by running scripts, not being able to use the GUI in SQL Server 2008 because I didn't have admin privileges.

Everything was moved without issue, however, when running a stored procedure on the new schema.table, I found I lost the identity field in a table. I double checked the script that created the table and it was there, however, SQL Server didn't get it when I ran the script. I was told later by a DBA that he had seen this same problem before.

In any event, for SQL Server 2008, these are the steps I took to get this resolved and they worked, so I'm posting this here in the hopes it will be a help to someone. This is what I did as I had FK dependencies on another table that made this more difficult:

I used this query to verify the identity was indeed missing and to view dependencies on the table.

1.) Find statistics on a table:

exec sp_help 'dbo.table_name_old';

2.) Create a duplicate, identical new table, except add an identity field on the PK field where it had been before.

3.) Disable the identity to move data.

SET IDENTITY_INSERT dbo.table_name ON 

4.) Transfer the data.

INSERT INTO dbo.table_name_new
(
field1, field2, etc...
)
SELECT 
field1, field2, etc...
FROM 
dbo.table_name_old;

5.) Verify the data is there.

SELECT * FROM dbo.table_name_new

6.) Re-enable the identity.

SET IDENTITY_INSERT ToyRecP.ToyAwards.lkpFile_New OFF

7.) This is the best script I found to get all the FK relationships to verify which table(s) the original table references as dependencies and I came across many, so it is a keeper!

SELECT f.name AS ForeignKey,
   OBJECT_NAME(f.parent_object_id) AS TableName,
   COL_NAME(fc.parent_object_id, fc.parent_column_id) AS ColumnName,
   OBJECT_NAME (f.referenced_object_id) AS ReferenceTableName,
   COL_NAME(fc.referenced_object_id, fc.referenced_column_id) AS ReferenceColumnName
FROM sys.foreign_keys AS f
INNER JOIN sys.foreign_key_columns AS fc
   ON f.OBJECT_ID = fc.constraint_object_id
   ORDER BY ReferenceTableName;

8.) Make sure you have all the PK and FK scripts for all the tables involved, before this next step.

9.) You can right-click on each key and script this using SQL Server 2008

10.) Drop the FK(s) from the dependency table(s) using this syntax:

ALTER TABLE [dbo].[table_name] DROP CONSTRAINT [Name_of_FK]

11.) Drop the original table:

DROP TABLE dbo.table_name_old;

13.) These next steps rely on the scripts you created in SQL Server 2008 in step 9.

--Add the PK to the new table.

--Add the FK to the new table.

--Add the FK's back to the dependency table.

14.) Verify everything is correct and complete. I used the GUI to look at the tables.

15.) Rename the new table to the original tables name.

exec sp_RENAME '[Schema_Name.OldTableName]' , '[NewTableName]';

Finally, everything worked!

T-SQL datetime rounded to nearest minute and nearest hours with using functions

I realize this question is ancient and there is an accepted and an alternate answer. I also realize that my answer will only answer half of the question, but for anyone wanting to round to the nearest minute and still have a datetime compatible value using only a single function:

CAST(YourValueHere as smalldatetime);

For hours or seconds, use Jeff Ogata's answer (the accepted answer) above.

List rows after specific date

Simply put:

SELECT * 
FROM TABLE_NAME
WHERE
dob > '1/21/2012'

Where 1/21/2012 is the date and you want all data, including that date.

SELECT * 
FROM TABLE_NAME
WHERE
dob BETWEEN '1/21/2012' AND '2/22/2012'

Use a between if you're selecting time between two dates

How to rollback or commit a transaction in SQL Server

The good news is a transaction in SQL Server can span multiple batches (each exec is treated as a separate batch.)

You can wrap your EXEC statements in a BEGIN TRANSACTION and COMMIT but you'll need to go a step further and rollback if any errors occur.

Ideally you'd want something like this:

BEGIN TRY
    BEGIN TRANSACTION 
        exec( @sqlHeader)
        exec(@sqlTotals)
        exec(@sqlLine)
    COMMIT
END TRY
BEGIN CATCH

    IF @@TRANCOUNT > 0
        ROLLBACK
END CATCH

The BEGIN TRANSACTION and COMMIT I believe you are already familiar with. The BEGIN TRY and BEGIN CATCH blocks are basically there to catch and handle any errors that occur. If any of your EXEC statements raise an error, the code execution will jump to the CATCH block.

Your existing SQL building code should be outside the transaction (above) as you always want to keep your transactions as short as possible.

Add column to SQL query results

why dont you add a "source" column to each of the queries with a static value like

select 'source 1' as Source, column1, column2...
from table1

UNION ALL

select 'source 2' as Source, column1, column2...
from table2

Import Excel spreadsheet columns into SQL Server database

You could use OPENROWSET, something like:

SELECT * FROM OPENROWSET('Microsoft.Jet.OLEDB.4.0', 
  'Excel 8.0;IMEX=1;HDR=NO;DATABASE=C:\FILE.xls', 'Select * from [Sheet1$]'

Just make sure the path is a path on the server, not your local machine.

Could not obtain information about Windows NT group user

I had to connect to VPN for the publish script to successfully deploy to the DB.

How to insert 1000 rows at a time

DECLARE @X INT = 1
WHILE @X <=1000
BEGIN
    INSERT INTO dbo.YourTable (ID, Age)
    VALUES(@X,LEFT(RAND()*100,2) 
    SET @X+=1
END;

    enter code here
DECLARE @X INT = 1
WHILE @X <=1000
BEGIN
    INSERT INTO dbo.YourTable (ID, Age)
    VALUES(@X,LEFT(RAND()*100,2) 
    SET @X+=1
END;

Control flow in T-SQL SP using IF..ELSE IF - are there other ways?

Also you can try to formulate your answer in the form of a SELECT CASE Statement. You can then later create simple if then's that use your results if needed as you have narrowed down the possibilities.

SELECT @Result =   
CASE @inputParam   
WHEN 1 THEN 1   
WHEN 2 THEN 2   
WHEN 3 THEN 1   
ELSE 4   
END  

IF @Result = 1   
BEGIN  
...  
END  

IF @Result = 2   
BEGIN   
....  
END  

IF @Result = 4   
BEGIN   
//Error handling code   
END   

Executing set of SQL queries using batch file?

Different ways:

  1. Using SQL Server Agent (If local instance)
    schedule a job in sql server agent with a new step having type as "T-SQL" then run the job.

  2. Using SQLCMD
    To use SQLCMD refer http://technet.microsoft.com/en-us/library/ms162773.aspx

  3. Using SQLPS
    To use SQLPS refer http://technet.microsoft.com/en-us/library/cc280450.aspx

Find duplicate records in a table using SQL Server

The following is running code:

SELECT abnno, COUNT(abnno)
FROM tbl_Name
GROUP BY abnno
HAVING ( COUNT(abnno) > 1 )

insert a NOT NULL column to an existing table

As an option you can initially create Null-able column, then update your table column with valid not null values and finally ALTER column to set NOT NULL constraint:

ALTER TABLE MY_TABLE ADD STAGE INT NULL
GO
UPDATE MY_TABLE SET <a valid not null values for your column>
GO
ALTER TABLE MY_TABLE ALTER COLUMN STAGE INT NOT NULL
GO

Another option is to specify correct default value for your column:

ALTER TABLE MY_TABLE ADD STAGE INT NOT NULL DEFAULT '0'

UPD: Please note that answer above contains GO which is a must when you run this code on Microsoft SQL server. If you want to perform the same operation on Oracle or MySQL you need to use semicolon ; like that:

ALTER TABLE MY_TABLE ADD STAGE INT NULL;
UPDATE MY_TABLE SET <a valid not null values for your column>;
ALTER TABLE MY_TABLE ALTER COLUMN STAGE INT NOT NULL;

Can I create a One-Time-Use Function in a Script or Stored Procedure?

I know I might get criticized for suggesting dynamic SQL, but sometimes it's a good solution. Just make sure you understand the security implications before you consider this.

DECLARE @add_a_b_func nvarchar(4000) = N'SELECT @c = @a + @b;';
DECLARE @add_a_b_parm nvarchar(500) = N'@a int, @b int, @c int OUTPUT';

DECLARE @result int;
EXEC sp_executesql @add_a_b_func, @add_a_b_parm, 2, 3, @c = @result OUTPUT;
PRINT CONVERT(varchar, @result); -- prints '5'

Listing information about all database files in SQL Server

You can also try this.

 select db_name(dbid) dbname, filename from sys.sysaltfiles

How to combine results of two queries into a single dataset

Try this:

SELECT ProductName,NumberofProducts ,NumberofProductssold
   FROM table1 
     JOIN table2
     ON table1.ProductName = table2.ProductName

Sql server - log is full due to ACTIVE_TRANSACTION

Here is what I ended up doing to work around the error.

First, I set up the database recovery model as SIMPLE. More information here.

Then, by deleting some old files I was able to make 5GB of free space which gave the log file more space to grow.

I reran the DELETE statement sucessfully without any warning.

I thought that by running the DELETE statement the database would inmediately become smaller thus freeing space in my hard drive. But that was not true. The space freed after a DELETE statement is not returned to the operating system inmediatedly unless you run the following command:

DBCC SHRINKDATABASE (MyDb, 0);
GO

More information about that command here.

SELECT query with CASE condition and SUM()

To get each sum in a separate column:

Select SUM(IF(CPaymentType='Check', CAmount, 0)) as PaymentAmountCheck,
       SUM(IF(CPaymentType='Cash', CAmount, 0)) as PaymentAmountCash
from TableOrderPayment
where CPaymentType IN ('Check','Cash') 
and CDate<=SYSDATETIME() 
and CStatus='Active';

DISTINCT for only one column

Try this:

SELECT ID, Email, ProductName, ProductModel FROM Products WHERE ID IN (SELECT MAX(ID) FROM Products GROUP BY Email)

Connecting to local SQL Server database using C#

You try with this string connection

Server=.\SQLExpress;AttachDbFilename=|DataDirectory|Database1.mdf;Database=dbname; Trusted_Connection=Yes;

Printing integer variable and string on same line in SQL

You may try this one,

declare @Number INT = 5                            
print 'There are ' + CONVERT(VARCHAR, @Number) + ' alias combinations did not match a record'

How to query values from xml nodes?

SELECT  b.BatchID,
        x.XmlCol.value('(ReportHeader/OrganizationReportReferenceIdentifier)[1]','VARCHAR(100)') AS OrganizationReportReferenceIdentifier,
        x.XmlCol.value('(ReportHeader/OrganizationNumber)[1]','VARCHAR(100)') AS OrganizationNumber
FROM    Batches b
CROSS APPLY b.RawXml.nodes('/CasinoDisbursementReportXmlFile/CasinoDisbursementReport') x(XmlCol);

Demo: SQLFiddle

Update a table using JOIN in SQL Server?

    UPDATE mytable
         SET myfield = CASE other_field
             WHEN 1 THEN 'value'
             WHEN 2 THEN 'value'
             WHEN 3 THEN 'value'
         END
    From mytable
    Join otherTable on otherTable.id = mytable.id
    Where othertable.somecolumn = '1234'

More alternatives here.

SQL Server: Null VS Empty String

An empty string is a string with zero length or no character. Null is absence of data.

Quick way to retrieve user information Active Directory

The reason why your code is slow is that your LDAP query retrieves every single user object in your domain even though you're only interested in one user with a common name of "Adit":

dSearcher.Filter = "(&(objectClass=user))";

So to optimize, you need to narrow your LDAP query to just the user you are interested in. Try something like:

dSearcher.Filter = "(&(objectClass=user)(cn=Adit))";

In addition, don't forget to dispose these objects when done:

  • DirectoryEntry dEntry
  • DirectorySearcher dSearcher

How can I preview a merge in git?

Adding to the existing answers, an alias could be created to show the diff and/or log prior to a merge. Many answers omit the fetch to be done first before "previewing" the merge; this is an alias that combines these two steps into one (emulating something similar to mercurial's hg incoming / outgoing)

So, building on "git log ..otherbranch", you can add the following to ~/.gitconfig :

...
[alias]
    # fetch and show what would be merged (use option "-p" to see patch)
    incoming = "!git remote update -p; git log ..@{u}"

For symmetry, the following alias can be used to show what is committed and would be pushed, prior to pushing:

    # what would be pushed (currently committed)
    outgoing = log @{u}..

And then you can run "git incoming" to show a lot of changes, or "git incoming -p" to show the patch (i.e., the "diff"), "git incoming --pretty=oneline", for a terse summary, etc. You may then (optionally) run "git pull" to actually merge. (Though, since you've already fetched, the merge could be done directly.)

Likewise, "git outgoing" shows what would be pushed if you were to run "git push".

How do I trim whitespace?

If using Python 3: In your print statement, finish with sep="". That will separate out all of the spaces.

EXAMPLE:

txt="potatoes"
print("I love ",txt,"",sep="")

This will print: I love potatoes.

Instead of: I love potatoes .

In your case, since you would be trying to get ride of the \t, do sep="\t"

Using lodash to compare jagged arrays (items existence without order)

You can use lodashs xor for this

doArraysContainSameElements = _.xor(arr1, arr2).length === 0

If you consider array [1, 1] to be different than array [1] then you may improve performance a bit like so:

doArraysContainSameElements = arr1.length === arr2.length === 0 && _.xor(arr1, arr2).length === 0

Spring MVC Missing URI template variable

@PathVariable is used to tell Spring that part of the URI path is a value you want passed to your method. Is this what you want, or are the variables supposed to be form data posted to the URI?

If you want form data, use @RequestParam instead of @PathVariable.

If you want @PathVariable, you need to specify placeholders in the @RequestMapping entry to tell Spring where the path variables fit in the URI. For example, if you want to extract a path variable called contentId, you would use:

@RequestMapping(value = "/whatever/{contentId}", method = RequestMethod.POST)

Edit: Additionally, if your path variable could contain a '.' and you want that part of the data, then you will need to tell Spring to grab everything, not just the stuff before the '.':

@RequestMapping(value = "/whatever/{contentId:.*}", method = RequestMethod.POST)

This is because the default behaviour of Spring is to treat that part of the URL as if it is a file extension, and excludes it from variable extraction.

Compiled vs. Interpreted Languages

The Python Book © 2015 Imagine Publishing Ltd, simply distunguishes the difference by the following hint mentioned in page 10 as:

An interpreted language such as Python is one where the source code is converted to machine code and then executed each time the program runs. This is different from a compiled language such as C, where the source code is only converted to machine code once – the resulting machine code is then executed each time the program runs.

How to copy JavaScript object to new variable NOT by reference?

I've found that the following works if you're not using jQuery and only interested in cloning simple objects (see comments).

JSON.parse(JSON.stringify(json_original));

Documentation

clk'event vs rising_edge()

rising_edge is defined as:

FUNCTION rising_edge  (SIGNAL s : std_ulogic) RETURN BOOLEAN IS
BEGIN
    RETURN (s'EVENT AND (To_X01(s) = '1') AND
                        (To_X01(s'LAST_VALUE) = '0'));
END;

FUNCTION To_X01  ( s : std_ulogic ) RETURN  X01 IS
BEGIN
    RETURN (cvt_to_x01(s));
END;

CONSTANT cvt_to_x01 : logic_x01_table := (
                     'X',  -- 'U'
                     'X',  -- 'X'
                     '0',  -- '0'
                     '1',  -- '1'
                     'X',  -- 'Z'
                     'X',  -- 'W'
                     '0',  -- 'L'
                     '1',  -- 'H'
                     'X'   -- '-'
                    );

If your clock only goes from 0 to 1, and from 1 to 0, then rising_edge will produce identical code. Otherwise, you can interpret the difference.

Personally, my clocks only go from 0 to 1 and vice versa. I find rising_edge(clk) to be more descriptive than the (clk'event and clk = '1') variant.

@AspectJ pointcut for all methods of a class with specific annotation

Using annotations, as described in the question.

Annotation: @Monitor

Annotation on class, app/PagesController.java:

package app;
@Controller
@Monitor
public class PagesController {
    @RequestMapping(value = "/", method = RequestMethod.GET)
    public @ResponseBody String home() {
        return "w00t!";
    }
}

Annotation on method, app/PagesController.java:

package app;
@Controller
public class PagesController {
    @Monitor
    @RequestMapping(value = "/", method = RequestMethod.GET)
    public @ResponseBody String home() {
        return "w00t!";
    }
}

Custom annotation, app/Monitor.java:

package app;
@Component
@Target(value = {ElementType.METHOD, ElementType.TYPE})
@Retention(value = RetentionPolicy.RUNTIME)
public @interface Monitor {
}

Aspect for annotation, app/MonitorAspect.java:

package app;
@Component
@Aspect
public class MonitorAspect {
    @Before(value = "@within(app.Monitor) || @annotation(app.Monitor)")
    public void before(JoinPoint joinPoint) throws Throwable {
        LogFactory.getLog(MonitorAspect.class).info("monitor.before, class: " + joinPoint.getSignature().getDeclaringType().getSimpleName() + ", method: " + joinPoint.getSignature().getName());
    }

    @After(value = "@within(app.Monitor) || @annotation(app.Monitor)")
    public void after(JoinPoint joinPoint) throws Throwable {
        LogFactory.getLog(MonitorAspect.class).info("monitor.after, class: " + joinPoint.getSignature().getDeclaringType().getSimpleName() + ", method: " + joinPoint.getSignature().getName());
    }
}

Enable AspectJ, servlet-context.xml:

<aop:aspectj-autoproxy />

Include AspectJ libraries, pom.xml:

<artifactId>spring-aop</artifactId>
<artifactId>aspectjrt</artifactId>
<artifactId>aspectjweaver</artifactId>
<artifactId>cglib</artifactId>

Pretty-Print JSON in Java

Now this can be achieved with the JSONLib library:

http://json-lib.sourceforge.net/apidocs/net/sf/json/JSONObject.html

If (and only if) you use the overloaded toString(int indentationFactor) method and not the standard toString() method.

I have verified this on the following version of the API:

<dependency>
  <groupId>org.json</groupId>
  <artifactId>json</artifactId>
  <version>20140107</version>
</dependency>

Parsing JSON using C

You can have a look at Jansson

The website states the following: Jansson is a C library for encoding, decoding and manipulating JSON data. It features:

  • Simple and intuitive API and data model
  • Can both encode to and decode from JSON
  • Comprehensive documentation
  • No dependencies on other libraries
  • Full Unicode support (UTF-8)
  • Extensive test suite

What is the JSF resource library for and how should it be used?

Actually, all of those examples on the web wherein the common content/file type like "js", "css", "img", etc is been used as library name are misleading.

Real world examples

To start, let's look at how existing JSF implementations like Mojarra and MyFaces and JSF component libraries like PrimeFaces and OmniFaces use it. No one of them use resource libraries this way. They use it (under the covers, by @ResourceDependency or UIViewRoot#addComponentResource()) the following way:

<h:outputScript library="javax.faces" name="jsf.js" />
<h:outputScript library="primefaces" name="jquery/jquery.js" />
<h:outputScript library="omnifaces" name="omnifaces.js" />
<h:outputScript library="omnifaces" name="fixviewstate.js" />
<h:outputScript library="omnifaces.combined" name="[dynamicname].js" />
<h:outputStylesheet library="primefaces" name="primefaces.css" />
<h:outputStylesheet library="primefaces-aristo" name="theme.css" />
<h:outputStylesheet library="primefaces-vader" name="theme.css" />

It should become clear that it basically represents the common library/module/theme name where all of those resources commonly belong to.

Easier identifying

This way it's so much easier to specify and distinguish where those resources belong to and/or are coming from. Imagine that you happen to have a primefaces.css resource in your own webapp wherein you're overriding/finetuning some default CSS of PrimeFaces; if PrimeFaces didn't use a library name for its own primefaces.css, then the PrimeFaces own one wouldn't be loaded, but instead the webapp-supplied one, which would break the look'n'feel.

Also, when you're using a custom ResourceHandler, you can also apply more finer grained control over resources coming from a specific library when library is used the right way. If all component libraries would have used "js" for all their JS files, how would the ResourceHandler ever distinguish if it's coming from a specific component library? Examples are OmniFaces CombinedResourceHandler and GraphicResourceHandler; check the createResource() method wherein the library is checked before delegating to next resource handler in chain. This way they know when to create CombinedResource or GraphicResource for the purpose.

Noted should be that RichFaces did it wrong. It didn't use any library at all and homebrewed another resource handling layer over it and it's therefore impossible to programmatically identify RichFaces resources. That's exactly the reason why OmniFaces CombinedResourceHander had to introduce a reflection-based hack in order to get it to work anyway with RichFaces resources.

Your own webapp

Your own webapp does not necessarily need a resource library. You'd best just omit it.

<h:outputStylesheet name="css/style.css" />
<h:outputScript name="js/script.js" />
<h:graphicImage name="img/logo.png" />

Or, if you really need to have one, you can just give it a more sensible common name, like "default" or some company name.

<h:outputStylesheet library="default" name="css/style.css" />
<h:outputScript library="default" name="js/script.js" />
<h:graphicImage library="default" name="img/logo.png" />

Or, when the resources are specific to some master Facelets template, you could also give it the name of the template, so that it's easier to relate each other. In other words, it's more for self-documentary purposes. E.g. in a /WEB-INF/templates/layout.xhtml template file:

<h:outputStylesheet library="layout" name="css/style.css" />
<h:outputScript library="layout" name="js/script.js" />

And a /WEB-INF/templates/admin.xhtml template file:

<h:outputStylesheet library="admin" name="css/style.css" />
<h:outputScript library="admin" name="js/script.js" />

For a real world example, check the OmniFaces showcase source code.

Or, when you'd like to share the same resources over multiple webapps and have created a "common" project for that based on the same example as in this answer which is in turn embedded as JAR in webapp's /WEB-INF/lib, then also reference it as library (name is free to your choice; component libraries like OmniFaces and PrimeFaces also work that way):

<h:outputStylesheet library="common" name="css/style.css" />
<h:outputScript library="common" name="js/script.js" />
<h:graphicImage library="common" name="img/logo.png" />

Library versioning

Another main advantage is that you can apply resource library versioning the right way on resources provided by your own webapp (this doesn't work for resources embedded in a JAR). You can create a direct child subfolder in the library folder with a name in the \d+(_\d+)* pattern to denote the resource library version.

WebContent
 |-- resources
 |    `-- default
 |         `-- 1_0
 |              |-- css
 |              |    `-- style.css
 |              |-- img
 |              |    `-- logo.png
 |              `-- js
 |                   `-- script.js
 :

When using this markup:

<h:outputStylesheet library="default" name="css/style.css" />
<h:outputScript library="default" name="js/script.js" />
<h:graphicImage library="default" name="img/logo.png" />

This will generate the following HTML with the library version as v parameter:

<link rel="stylesheet" type="text/css" href="/contextname/javax.faces.resource/css/style.css.xhtml?ln=default&amp;v=1_0" />
<script type="text/javascript" src="/contextname/javax.faces.resource/js/script.js.xhtml?ln=default&amp;v=1_0"></script>
<img src="/contextname/javax.faces.resource/img/logo.png.xhtml?ln=default&amp;v=1_0" alt="" />

So, if you have edited/updated some resource, then all you need to do is to copy or rename the version folder into a new value. If you have multiple version folders, then the JSF ResourceHandler will automatically serve the resource from the highest version number, according to numerical ordering rules.

So, when copying/renaming resources/default/1_0/* folder into resources/default/1_1/* like follows:

WebContent
 |-- resources
 |    `-- default
 |         |-- 1_0
 |         |    :
 |         |
 |         `-- 1_1
 |              |-- css
 |              |    `-- style.css
 |              |-- img
 |              |    `-- logo.png
 |              `-- js
 |                   `-- script.js
 :

Then the last markup example would generate the following HTML:

<link rel="stylesheet" type="text/css" href="/contextname/javax.faces.resource/css/style.css.xhtml?ln=default&amp;v=1_1" />
<script type="text/javascript" src="/contextname/javax.faces.resource/js/script.js.xhtml?ln=default&amp;v=1_1"></script>
<img src="/contextname/javax.faces.resource/img/logo.png.xhtml?ln=default&amp;v=1_1" alt="" />

This will force the webbrowser to request the resource straight from the server instead of showing the one with the same name from the cache, when the URL with the changed parameter is been requested for the first time. This way the endusers aren't required to do a hard refresh (Ctrl+F5 and so on) when they need to retrieve the updated CSS/JS resource.

Please note that library versioning is not possible for resources enclosed in a JAR file. You'd need a custom ResourceHandler. See also How to use JSF versioning for resources in jar.

See also:

How to read file contents into a variable in a batch file?

You can read multiple variables from file like this:

for /f "delims== tokens=1,2" %%G in (param.txt) do set %%G=%%H

where param.txt:

PARAM1=value1
PARAM2=value2
...

How to print Unicode character in C++?

'1060' is four characters, and won't compile under the standard. You should just treat the character as a number, if your wide characters match 1:1 with Unicode (check your locale settings).

int main (){
    wchar_t f = 1060;
    wcout << f << endl;
}

Adding line break in C# Code behind page

 result = "Minimum MarketData"+ Environment.NewLine
           + "Refresh interval is 1";

What is the difference between Cloud Computing and Grid Computing?

A Grid is a hardware and software infrastructure that clusters and integrates high-end computers, networks, databases, and scientific instruments from multiple sources to form a virtual supercomputer on which users can work collaboratively within virtual organisations

Grid is Mostly free used by academic research etc.

Clouds are a large pool of easily usable and accessible virtualized resources (such as hardware, development platforms and/or services). These resources can be dynamically reconfigured to adjust to a variable load (scale), allowing also for an optimum resource utilization. This pool of resources is typically exploited by a pay peruse model in which guarantees are offered by the Infrastructure Provider by customized service level agreements.

Cloud is not free. It is a service, provided by different service providers and they charge according to your work done.

CSS last-child(-1)

Unless you can get PHP to label that element with a class you are better to use jQuery.

jQuery(document).ready(function () {
  $count =  jQuery("ul li").size() - 1;
  alert($count);
  jQuery("ul li:nth-child("+$count+")").css("color","red");
});

Arrays in unix shell?

If you want a key value store with support for spaces use the -A parameter:

declare -A programCollection
programCollection["xwininfo"]="to aquire information about the target window."

for program in ${!programCollection[@]}
do
    echo "The program ${program} is used ${programCollection[${program}]}"
done

http://linux.die.net/man/1/bash "Associative arrays are created using declare -A name. "

How to set cursor to input box in Javascript?

You have not provided enough code to help You likely submit the form and reload the page OR you have an object on the page like an embedded PDF that steals the focus.

Here is the canonical plain javascript method of validating a form It can be improved with onubtrusive JS which will remove the inline script, but this is the starting point DEMO

function validate(formObj) {
  document.getElementById("errorMsg").innerHTML = "";    
  var quantity = formObj.quantity;
  if (isNaN(quantity)) {
    quantity.value="";
    quantity.focus();
    document.getElementById("errorMsg").innerHTML = "Only numeric value is allowed";
    return false;
  }
  return true; // allow submit
}   

Here is the HTML

<form onsubmit="return validate(this)">
    <input type="text" name="quantity" value="" />
    <input type="submit" />
</form>    
<span id="errorMsg"></span>

Passing parameters to addTarget:action:forControlEvents

This fixed my problem but it crashed unless I changed

action:@selector(switchToNewsDetails:event:)                 

to

action:@selector(switchToNewsDetails: forEvent:)              

How can I send a file document to the printer and have it print?

    public static void PrintFileToDefaultPrinter(string FilePath)
    {
        try
        {
            var file = File.ReadAllBytes(FilePath);
            var printQueue = LocalPrintServer.GetDefaultPrintQueue();

            using (var job = printQueue.AddJob())
            using (var stream = job.JobStream)
            {
                stream.Write(file, 0, file.Length);
            }
        }
        catch (Exception)
        {

            throw;
        }
    }

Difference between malloc and calloc?

malloc() and calloc() are functions from the C standard library that allow dynamic memory allocation, meaning that they both allow memory allocation during runtime.

Their prototypes are as follows:

void *malloc( size_t n);
void *calloc( size_t n, size_t t)

There are mainly two differences between the two:

  • Behavior: malloc() allocates a memory block, without initializing it, and reading the contents from this block will result in garbage values. calloc(), on the other hand, allocates a memory block and initializes it to zeros, and obviously reading the content of this block will result in zeros.

  • Syntax: malloc() takes 1 argument (the size to be allocated), and calloc() takes two arguments (number of blocks to be allocated and size of each block).

The return value from both is a pointer to the allocated block of memory, if successful. Otherwise, NULL will be returned indicating the memory allocation failure.

Example:

int *arr;

// allocate memory for 10 integers with garbage values
arr = (int *)malloc(10 * sizeof(int)); 

// allocate memory for 10 integers and sets all of them to 0
arr = (int *)calloc(10, sizeof(int));

The same functionality as calloc() can be achieved using malloc() and memset():

// allocate memory for 10 integers with garbage values   
arr= (int *)malloc(10 * sizeof(int));
// set all of them to 0
memset(arr, 0, 10 * sizeof(int)); 

Note that malloc() is preferably used over calloc() since it's faster. If zero-initializing the values is wanted, use calloc() instead.

How can I disable the UITableView selection?

You Can also set the background color to Clear to achieve the same effect as UITableViewCellSelectionStyleNone, in case you don't want to/ can't use UITableViewCellSelectionStyleNone.

You would use code like the following:

UIView *backgroundColorView = [[UIView alloc] init];
backgroundColorView.backgroundColor = [UIColor clearColor];
backgroundColorView.layer.masksToBounds = YES;
[cell setSelectedBackgroundView: backgroundColorView];

This may degrade your performance as your adding an extra colored view to each cell.

What are .NET Assemblies?

An assembly is a collection of types and resources that forms a logical unit of functionality. All types in the .NET Framework must exist in assemblies; the common language runtime does not support types outside of assemblies. Each time you create a Microsoft Windows® Application, Windows Service, Class Library, or other application with Visual Basic .NET, you're building a single assembly. Each assembly is stored as an .exe or .dll file.

Source : https://msdn.microsoft.com/en-us/library/ms973231.aspx#assenamesp_topic4

For those with Java background like me hope following diagram clarifies concepts -

Assemblies are just like jar files (containing multiple .class files). Your code can reference an existing assemblie or you code itself can be published as an assemblie for other code to reference and use (you can think this as jar files in Java that you can add in your project dependencies).

At the end of the day an assembly is a compiled code that can be run on any operating system with CLR installed. This is same as saying .class file or bundled jar can run on any machine with JVM installed.

enter image description here

Spring MVC: How to perform validation?

If you have same error handling logic for different method handlers, then you would end up with lots of handlers with following code pattern:

if (validation.hasErrors()) {
  // do error handling
}
else {
  // do the actual business logic
}

Suppose you're creating RESTful services and want to return 400 Bad Request along with error messages for every validation error case. Then, the error handling part would be same for every single REST endpoint that requires validation. Repeating that very same logic in every single handler is not so DRYish!

One way to solve this problem is to drop the immediate BindingResult after each To-Be-Validated bean. Now, your handler would be like this:

@RequestMapping(...)
public Something doStuff(@Valid Somebean bean) { 
    // do the actual business logic
    // Just the else part!
}

This way, if the bound bean was not valid, a MethodArgumentNotValidException will be thrown by Spring. You can define a ControllerAdvice that handles this exception with that same error handling logic:

@ControllerAdvice
public class ErrorHandlingControllerAdvice {
    @ExceptionHandler(MethodArgumentNotValidException.class)
    public SomeErrorBean handleValidationError(MethodArgumentNotValidException ex) {
        // do error handling
        // Just the if part!
    }
}

You still can examine the underlying BindingResult using getBindingResult method of MethodArgumentNotValidException.

Where in an Eclipse workspace is the list of projects stored?

You can also have several workspaces - so you can connect to one and have set "A" of projects - and then connect to a different set when ever you like.

Why are arrays of references illegal?

You can get fairly close with this template struct. However, you need to initialize with expressions that are pointers to T, rather than T; and so, though you can easily make a 'fake_constref_array' similarly, you won't be able to bind that to rvalues as done in the OP's example ('8');

#include <stdio.h>

template<class T, int N> 
struct fake_ref_array {
   T * ptrs[N];
  T & operator [] ( int i ){ return *ptrs[i]; }
};

int A,B,X[3];

void func( int j, int k)
{
  fake_ref_array<int,3> refarr = { &A, &B, &X[1] };
  refarr[j] = k;  // :-) 
   // You could probably make the following work using an overload of + that returns
   // a proxy that overloads *. Still not a real array though, so it would just be
   // stunt programming at that point.
   // *(refarr + j) = k  
}

int
main()
{
    func(1,7);  //B = 7
    func(2,8);     // X[1] = 8
    printf("A=%d B=%d X = {%d,%d,%d}\n", A,B,X[0],X[1],X[2]);
        return 0;
}

--> A=0 B=7 X = {0,8,0}

Java Desktop application: SWT vs. Swing

If you plan to build a full functional applications with more than a handful of features, I will suggest to jump right to using Eclipse RCP as the framework.

If your application won't grow too big or your requirements are just too unique to be handled by a normal business framework, you can safely jump with Swing.

At the end of the day, I'd suggest you to try both technologies to find the one suit you better. Like Netbeans vs Eclipse vs IntelliJ, there is no the absolute correct answer here and both frameworks have their own drawbacks.

Pro Swing:

  • more experts
  • more Java-like (almost no public field, no need to dispose on resource)

Pro SWT:

  • more OS native
  • faster

Truncate string in Laravel blade templates

For simple things like this I would prefer to make a helper - for example:

create a helpers.php file in your /app/helpers.php with following content:

<?php
if (! function_exists('short_string')) {
    function short_string($str) {
            $rest = substr($str, 0, 10);
            return $rest;
    }
}

Register the helper.php at autoload in your composer.json

   "autoload": {
        "files": [
            "app/helpers.php"
        ],
        "psr-4": {
            "App\\": "app/"
        },
        "classmap": [
            "database/seeds",
            "database/factories"
        ]
    }

After that you can use in your blade file for example:

{{ short_string($whatever_as_text) }}

You can use this simple function, then, globally in your app.

Sum the digits of a number

you can also try this with built_in_function called divmod() ;

number = int(input('enter any integer: = '))
sum = 0
while number!=0: 
    take = divmod(number, 10) 
    dig = take[1] 
    sum += dig 
    number = take[0] 
print(sum) 

you can take any number of digit

How to get the 'height' of the screen using jquery

$(window).height();   // returns height of browser viewport
$(document).height(); // returns height of HTML document

As documented here: http://api.jquery.com/height/

Passing data between different controller action methods

If you need to pass data from one controller to another you must pass data by route values.Because both are different request.if you send data from one page to another then you have to user query string(same as route values).

But you can do one trick :

In your calling action call the called action as a simple method :

public class ServerController : Controller
{
 [HttpPost]
 public ActionResult ApplicationPoolsUpdate(ServiceViewModel viewModel)
 {
      XDocument updatedResultsDocument = myService.UpdateApplicationPools();
      ApplicationPoolController pool=new ApplicationPoolController(); //make an object of ApplicationPoolController class.

      return pool.UpdateConfirmation(updatedResultsDocument); // call the ActionMethod you want as a simple method and pass the model as an argument.
      // Redirect to ApplicationPool controller and pass
      // updatedResultsDocument to be used in UpdateConfirmation action method
 }
}

How do I horizontally center an absolute positioned element inside a 100% width div?

Was missing the use of calc in the answers, which is a cleaner solution.

#logo {
  position: absolute;
  left: calc(50% - 25px);
  height: 50px;
  width: 50px;
  background: red;
}

jsFiddle

Works in most modern browsers: http://caniuse.com/calc

Maybe it's too soon to use it without a fallback, but I thought maybe for future visitors it would be helpful.

Text was truncated or one or more characters had no match in the target code page including the primary key in an unpivot

None of the above worked for me. I SOLVED my problem by saving my source data (save as) Excel file as a single xls Worksheet Excel 5.0/95 and imported without column headings. Also, I created the table in advance and mapped manually instead of letting SQL create the table.

Using local makefile for CLion instead of CMake

Update: If you are using CLion 2020.2, then it already supports Makefiles. If you are using an older version, read on.


Even though currently only CMake is supported, you can instruct CMake to call make with your custom Makefile. Edit your CMakeLists.txt adding one of these two commands:

When you tell CLion to run your program, it will try to find an executable with the same name of the target in the directory pointed by PROJECT_BINARY_DIR. So as long as your make generates the file where CLion expects, there will be no problem.

Here is a working example:

Tell CLion to pass its $(PROJECT_BINARY_DIR) to make

This is the sample CMakeLists.txt:

cmake_minimum_required(VERSION 2.8.4)
project(mytest)

set(CMAKE_CXX_FLAGS "${CMAKE_CXX_FLAGS} -std=c++11")

add_custom_target(mytest COMMAND make -C ${mytest_SOURCE_DIR}
                         CLION_EXE_DIR=${PROJECT_BINARY_DIR})

Tell make to generate the executable in CLion's directory

This is the sample Makefile:

all:
    echo Compiling $(CLION_EXE_DIR)/$@ ...
    g++ mytest.cpp -o $(CLION_EXE_DIR)/mytest

That is all, you may also want to change your program's working directory so it executes as it is when you run make from inside your directory. For this edit: Run -> Edit Configurations ... -> mytest -> Working directory

Is it possible to auto-format your code in Dreamweaver?

Coding toolbar (Edit > Toolbars > Coding) and select Code Format Settings to set your preferred formatting.

You can also access the formatting option from Commands > Apply Source Formatting or only apply it to a select block of code by selecting the Apply Source Formatting to Selection option.

http://www.hongkiat.com/blog/10-useful-dreamweaver-tips-for-beginners/

Convert string to Color in C#

I've used something like this before:

        public static T CreateFromString<T>(string stringToCreateFrom) {

        T output = Activator.CreateInstance<T>();

        if (!output.GetType().IsEnum)
            throw new IllegalTypeException();

        try {
            output = (T) Enum.Parse(typeof (T), stringToCreateFrom, true);
        }
        catch (Exception ex) {
            string error = "Cannot parse '" + stringToCreateFrom + "' to enum '" + typeof (T).FullName + "'";
            _logger.Error(error, ex);
            throw new IllegalArgumentException(error, ex);
        }

        return output;
    }

What's the difference between getRequestURI and getPathInfo methods in HttpServletRequest?

I will put a small comparison table here (just to have it somewhere):

Servlet is mapped as /test%3F/* and the application is deployed under /app.

http://30thh.loc:8480/app/test%3F/a%3F+b;jsessionid=S%3F+ID?p+1=c+d&p+2=e+f#a

Method              URL-Decoded Result           
----------------------------------------------------
getContextPath()        no      /app
getLocalAddr()                  127.0.0.1
getLocalName()                  30thh.loc
getLocalPort()                  8480
getMethod()                     GET
getPathInfo()           yes     /a?+b
getProtocol()                   HTTP/1.1
getQueryString()        no      p+1=c+d&p+2=e+f
getRequestedSessionId() no      S%3F+ID
getRequestURI()         no      /app/test%3F/a%3F+b;jsessionid=S+ID
getRequestURL()         no      http://30thh.loc:8480/app/test%3F/a%3F+b;jsessionid=S+ID
getScheme()                     http
getServerName()                 30thh.loc
getServerPort()                 8480
getServletPath()        yes     /test?
getParameterNames()     yes     [p 2, p 1]
getParameter("p 1")     yes     c d

In the example above the server is running on the localhost:8480 and the name 30thh.loc was put into OS hosts file.

Comments

  • "+" is handled as space only in the query string

  • Anchor "#a" is not transferred to the server. Only the browser can work with it.

  • If the url-pattern in the servlet mapping does not end with * (for example /test or *.jsp), getPathInfo() returns null.

If Spring MVC is used

  • Method getPathInfo() returns null.

  • Method getServletPath() returns the part between the context path and the session ID. In the example above the value would be /test?/a?+b

  • Be careful with URL encoded parts of @RequestMapping and @RequestParam in Spring. It is buggy (current version 3.2.4) and is usually not working as expected.

Oracle DateTime in Where Clause?

You can also use the following to include the TIME portion in your query:

SELECT EMP_NAME
     , DEPT
  FROM EMPLOYEE 
 WHERE TIME_CREATED >= TO_DATE('26/JAN/2011 00:00:00', 'dd/mon/yyyy HH24:MI:SS');

How to iterate over a std::map full of strings in C++

iter->first and iter->second are variables, you are attempting to call them as methods.

Simplest code for array intersection in javascript

function getIntersection(arr1, arr2){
    var result = [];
    arr1.forEach(function(elem){
        arr2.forEach(function(elem2){
            if(elem === elem2){
                result.push(elem);
            }
        });
    });
    return result;
}

getIntersection([1,2,3], [2,3,4,5]); // [ 2, 3 ]

How to clear the interpreter console?

As you mentioned, you can do a system call:

For Windows:

>>> import os
>>> clear = lambda: os.system('cls')
>>> clear()

For Linux it would be:

>>> import os
>>> clear = lambda: os.system('clear')
>>> clear()

How can I create persistent cookies in ASP.NET?

You need to add this as the last line...

HttpContext.Current.Response.Cookies.Add(userid);

When you need to read the value of the cookie, you'd use a method similar to this:

    string cookieUserID= String.Empty;

    try
    {
        if (HttpContext.Current.Request.Cookies["userid"] != null)
        {
            cookieUserID = HttpContext.Current.Request.Cookies["userid"];
        }
    }
    catch (Exception ex)
    {
       //handle error
    }

    return cookieUserID;

AngularJS : When to use service instead of factory

The concept for all these providers is much simpler than it initially appears. If you dissect a provider you and pull out the different parts it becomes very clear.

To put it simply each one of these providers is a specialized version of the other, in this order: provider > factory > value / constant / service.

So long the provider does what you can you can use the provider further down the chain which would result in writing less code. If it doesn't accomplish what you want you can go up the chain and you'll just have to write more code.

This image illustrates what I mean, in this image you will see the code for a provider, with the portions highlighted showing you which portions of the provider could be used to create a factory, value, etc instead.

AngularJS providers, factories, services, etc are all the same thing
(source: simplygoodcode.com)

For more details and examples from the blog post where I got the image from go to: http://www.simplygoodcode.com/2015/11/the-difference-between-service-provider-and-factory-in-angularjs/

What is the logic behind the "using" keyword in C++?

In C++11, the using keyword when used for type alias is identical to typedef.

7.1.3.2

A typedef-name can also be introduced by an alias-declaration. The identifier following the using keyword becomes a typedef-name and the optional attribute-specifier-seq following the identifier appertains to that typedef-name. It has the same semantics as if it were introduced by the typedef specifier. In particular, it does not define a new type and it shall not appear in the type-id.

Bjarne Stroustrup provides a practical example:

typedef void (*PFD)(double);    // C style typedef to make `PFD` a pointer to a function returning void and accepting double
using PF = void (*)(double);    // `using`-based equivalent of the typedef above
using P = [](double)->void; // using plus suffix return type, syntax error
using P = auto(double)->void // Fixed thanks to DyP

Pre-C++11, the using keyword can bring member functions into scope. In C++11, you can now do this for constructors (another Bjarne Stroustrup example):

class Derived : public Base { 
public: 
    using Base::f;    // lift Base's f into Derived's scope -- works in C++98
    void f(char);     // provide a new f 
    void f(int);      // prefer this f to Base::f(int) 

    using Base::Base; // lift Base constructors Derived's scope -- C++11 only
    Derived(char);    // provide a new constructor 
    Derived(int);     // prefer this constructor to Base::Base(int) 
    // ...
}; 

Ben Voight provides a pretty good reason behind the rationale of not introducing a new keyword or new syntax. The standard wants to avoid breaking old code as much as possible. This is why in proposal documents you will see sections like Impact on the Standard, Design decisions, and how they might affect older code. There are situations when a proposal seems like a really good idea but might not have traction because it would be too difficult to implement, too confusing, or would contradict old code.


Here is an old paper from 2003 n1449. The rationale seems to be related to templates. Warning: there may be typos due to copying over from PDF.

First let’s consider a toy example:

template <typename T>
class MyAlloc {/*...*/};

template <typename T, class A>
class MyVector {/*...*/};

template <typename T>

struct Vec {
typedef MyVector<T, MyAlloc<T> > type;
};
Vec<int>::type p; // sample usage

The fundamental problem with this idiom, and the main motivating fact for this proposal, is that the idiom causes the template parameters to appear in non-deducible context. That is, it will not be possible to call the function foo below without explicitly specifying template arguments.

template <typename T> void foo (Vec<T>::type&);

So, the syntax is somewhat ugly. We would rather avoid the nested ::type We’d prefer something like the following:

template <typename T>
using Vec = MyVector<T, MyAlloc<T> >; //defined in section 2 below
Vec<int> p; // sample usage

Note that we specifically avoid the term “typedef template” and introduce the new syntax involving the pair “using” and “=” to help avoid confusion: we are not defining any types here, we are introducing a synonym (i.e. alias) for an abstraction of a type-id (i.e. type expression) involving template parameters. If the template parameters are used in deducible contexts in the type expression then whenever the template alias is used to form a template-id, the values of the corresponding template parameters can be deduced – more on this will follow. In any case, it is now possible to write generic functions which operate on Vec<T> in deducible context, and the syntax is improved as well. For example we could rewrite foo as:

template <typename T> void foo (Vec<T>&);

We underscore here that one of the primary reasons for proposing template aliases was so that argument deduction and the call to foo(p) will succeed.


The follow-up paper n1489 explains why using instead of using typedef:

It has been suggested to (re)use the keyword typedef — as done in the paper [4] — to introduce template aliases:

template<class T> 
    typedef std::vector<T, MyAllocator<T> > Vec;

That notation has the advantage of using a keyword already known to introduce a type alias. However, it also displays several disavantages among which the confusion of using a keyword known to introduce an alias for a type-name in a context where the alias does not designate a type, but a template; Vec is not an alias for a type, and should not be taken for a typedef-name. The name Vec is a name for the family std::vector< [bullet] , MyAllocator< [bullet] > > – where the bullet is a placeholder for a type-name. Consequently we do not propose the “typedef” syntax. On the other hand the sentence

template<class T>
    using Vec = std::vector<T, MyAllocator<T> >;

can be read/interpreted as: from now on, I’ll be using Vec<T> as a synonym for std::vector<T, MyAllocator<T> >. With that reading, the new syntax for aliasing seems reasonably logical.

I think the important distinction is made here, aliases instead of types. Another quote from the same document:

An alias-declaration is a declaration, and not a definition. An alias- declaration introduces a name into a declarative region as an alias for the type designated by the right-hand-side of the declaration. The core of this proposal concerns itself with type name aliases, but the notation can obviously be generalized to provide alternate spellings of namespace-aliasing or naming set of overloaded functions (see ? 2.3 for further discussion). [My note: That section discusses what that syntax can look like and reasons why it isn't part of the proposal.] It may be noted that the grammar production alias-declaration is acceptable anywhere a typedef declaration or a namespace-alias-definition is acceptable.

Summary, for the role of using:

  • template aliases (or template typedefs, the former is preferred namewise)
  • namespace aliases (i.e., namespace PO = boost::program_options and using PO = ... equivalent)
  • the document says A typedef declaration can be viewed as a special case of non-template alias-declaration. It's an aesthetic change, and is considered identical in this case.
  • bringing something into scope (for example, namespace std into the global scope), member functions, inheriting constructors

It cannot be used for:

int i;
using r = i; // compile-error

Instead do:

using r = decltype(i);

Naming a set of overloads.

// bring cos into scope
using std::cos;

// invalid syntax
using std::cos(double);

// not allowed, instead use Bjarne Stroustrup function pointer alias example
using test = std::cos(double);

Android Layout Animations from bottom to top and top to bottom on ImageView click

create directory in /res/anim and create bottom_to_original.xml

<?xml version="1.0" encoding="utf-8"?>
<set xmlns:android="http://schemas.android.com/apk/res/android">
    <translate
        android:duration="1500"
        android:fromYDelta="100%"
        android:toYDelta="1%" />
</set>

JAVA:

    LinearLayout ll = findViewById(R.id.ll);

    Animation animation;
    animation = AnimationUtils.loadAnimation(getApplicationContext(),
            R.anim.sample_animation);
    ll .setAnimation(animation);

Python 2.7: %d, %s, and float()

See String Formatting Operations:

%d is the format code for an integer. %f is the format code for a float.

%s prints the str() of an object (What you see when you print(object)).

%r prints the repr() of an object (What you see when you print(repr(object)).

For a float %s, %r and %f all display the same value, but that isn't the case for all objects. The other fields of a format specifier work differently as well:

>>> print('%10.2s' % 1.123) # print as string, truncate to 2 characters in a 10-place field.
        1.
>>> print('%10.2f' % 1.123) # print as float, round to 2 decimal places in a 10-place field.
      1.12

What should I do when 'svn cleanup' fails?

$ ls -la .svn
$ rm -f .svn/lock

Then

$ svn update

Hope it helps

Android Studio Image Asset Launcher Icon Background Color

With "Asset Type" set to "Image", try setting the same image for the foreground and background layers, keeping the same "Resize" percentage.

Pure CSS scroll animation

And for webkit enabled browsers I've had good results with:

.myElement {
    -webkit-overflow-scrolling: touch;
    scroll-behavior: smooth; // Added in from answer from Felix
    overflow-x: scroll;
}

This makes scrolling behave much more like the standard browser behavior - at least it works well on the iPhone we were testing on!

Hope that helps,

Ed

How to add a class with React.js?

Taken from their site.

render() {
  let className = 'menu';
  if (this.props.isActive) {
    className += ' menu-active';
  }
  return <span className={className}>Menu</span>
}

https://reactjs.org/docs/faq-styling.html

How do I get the YouTube video ID from a URL?

function youtube_parser(url){
    var match = url.match(/^.*((youtu.be\/)|(v\/)|(\/u\/\w\/)|(embed\/)|(watch\?))\??v?=?([^#\&\?]*).*/);
    return (match&&match[7].length==11)?match[7]:false;
}

Shortest and Efficient

Inserting a string into a list without getting split into characters

I suggest to add the '+' operator as follows:

list = list + ['foo']

Hope it helps!

Use nginx to serve static files from subdirectories of a given directory

It should work, however http://nginx.org/en/docs/http/ngx_http_core_module.html#alias says:

When location matches the last part of the directive’s value: it is better to use the root directive instead:

which would yield:

server {
  listen        8080;
  server_name   www.mysite.com mysite.com;
  error_log     /home/www-data/logs/nginx_www.error.log;
  error_page    404    /404.html;

  location /public/doc/ {
    autoindex on;
    root  /home/www-data/mysite;
  } 

  location = /404.html {
    root /home/www-data/mysite/static/html;
  }       
}

Remove a CLASS for all child elements

You can also do like this :

  $("#table-filters li").parent().find('li').removeClass("active");

How do I get AWS_ACCESS_KEY_ID for Amazon?

It is very dangerous to create an access_key_id in "My Account ==> Security Credentials". Because the key has all authority. Please create "IAM" user and attach only some policies you need.

Run on server option not appearing in Eclipse

Do you see any servers in server view in eclipse? Probably simply you have not created any server instances.

How to convert dataframe into time series?

With library fpp, you can easily create time series with date format: time_ser=ts(data,frequency=4,start=c(1954,2))

here we start at the 2nd quarter of 1954 with quarter fequency.

Razor If/Else conditional operator syntax

You need to put the entire ternary expression in parenthesis. Unfortunately that means you can't use "@:", but you could do something like this:

@(deletedView ? "Deleted" : "Created by")

Razor currently supports a subset of C# expressions without using @() and unfortunately, ternary operators are not part of that set.

How to exit from the application and show the home screen?

There is another option, to use the FinishAffinity method to close all the tasks in the stack related to the app.

See: https://stackoverflow.com/a/27765687/1984636

Always show vertical scrollbar in <select>

add:

overflow-y: scroll

in your css bud.

In Maven how to exclude resources from the generated jar?

Another possibility is to use the Maven Shade Plugin, e.g. to exclude a logging properties file used only locally in your IDE:

<plugin>
    <groupId>org.apache.maven.plugins</groupId>
    <artifactId>maven-shade-plugin</artifactId>
    <version>${maven-shade-plugin-version}</version>
    <executions>
        <execution>
            <phase>package</phase>
            <goals>
                <goal>shade</goal>
            </goals>
            <configuration>
                <filters>
                    <filter>
                        <artifact>*:*</artifact>
                        <excludes>
                            <exclude>log4j2.xml</exclude>
                        </excludes>
                    </filter>
                </filters>
            </configuration>
        </execution>
    </executions>
</plugin>

This will however exclude the files from every artifact, so it might not be feasible in every situation.

How to get today's Date?

Code To Get Today's date in any specific Format

You can define the desired format in SimpleDateFormat instance to get the date in that specific formate

 DateFormat dateFormat = new SimpleDateFormat("MM/dd/yyyy");
        Calendar cal = Calendar.getInstance();
        Date date = cal.getTime();
        String todaysdate = dateFormat.format(date);
         System.out.println("Today's date : " + todaysdate);

Follow below links to see the valid date format combination.

https://docs.oracle.com/javase/7/docs/api/java/text/SimpleDateFormat.html

CODE : To get x days ahead or Previous from Today's date, To get the past date or Future date.

For Example :

Today date : 11/27/2018

xdayFromTodaysDate = 2 to get date as 11/29/2018

xdayFromTodaysDate = -2 to get date as 11/25/2018

  public String getAniversaryDate(int xdayFromTodaysDate ){

        DateFormat dateFormat = new SimpleDateFormat("MM/dd/yyyy");
        Calendar cal = Calendar.getInstance();
        Date date = cal.getTime();
        cal.setTime(date);
        cal.add(Calendar.DAY_OF_MONTH,xdayFromTodaysDate);
        date = cal.getTime();
        String aniversaryDate = dateFormat.format(date);
        LOGGER.info("Today's date : " + todaysdate);
          return aniversaryDate;

    }

CODE : To get x days ahead or Previous from a Given date

  public String getAniversaryDate(String givendate, int xdayFromTodaysDate ){


        Calendar cal = Calendar.getInstance();
        DateFormat dateFormat = new SimpleDateFormat("MM/dd/yyyy");
        try {
            Date date = dateFormat.parse(givendate);
            cal.setTime(date);
            cal.add(Calendar.DAY_OF_MONTH,xdayFromTodaysDate);
            date = cal.getTime();
            String aniversaryDate = dateFormat.format(date);
            LOGGER.info("aniversaryDate : " + aniversaryDate);
            return aniversaryDate;
        } catch (ParseException e) {
            e.printStackTrace();
            return null;
        }

    }

In android app Toolbar.setTitle method has no effect – application name is shown as title

If your goal is to set a static string in the toolbar, the easiest way to do it is to simply set the activity label in AndroidManifest.xml:

<activity android:name=".xxxxActivity"
          android:label="@string/string_id" />

The toolbar will get this string without any code. (works for me with v27 libraries.)

Struct Constructor in C++?

Yes. A structure is just like a class, but defaults to public:, in the class definition and when inheriting:

struct Foo
{
    int bar;

    Foo(void) :
    bar(0)
    {
    }
}

Considering your other question, I would suggest you read through some tutorials. They will answer your questions faster and more complete than we will.

T-SQL stored procedure that accepts multiple Id values

Erland Sommarskog has maintained the authoritative answer to this question for the last 16 years: Arrays and Lists in SQL Server.

There are at least a dozen ways to pass an array or list to a query; each has their own unique pros and cons.

I really can't recommend enough to read the article to learn about the tradeoffs among all these options.

No 'Access-Control-Allow-Origin' header is present on the requested resource. Origin '...' is therefore not allowed access

You need to add this at start of your php page "login.php"

<?php header('Access-Control-Allow-Origin: *'); ?>

What's the best way to store Phone number in Django models

It all depends in what you understand as phone number. Phone numbers are country specific. The localflavors packages for several countries contains their own "phone number field". So if you are ok being country specific you should take a look at localflavor package (class us.models.PhoneNumberField for US case, etc.)

Otherwise you could inspect the localflavors to get the maximun lenght for all countries. Localflavor also has forms fields you could use in conjunction with the country code to validate the phone number.

Delete rows from multiple tables using a single query (SQL Express 2005) with a WHERE condition

CREATE PROCEDURE sp_deleteUserDetails
    @Email varchar(255)
AS
    declare @tempRegId as int
    Delete UserRegistration where Email=@Email  
    set @tempRegId = (select Id from UserRegistration where Email = @Email)
    Delete UserProfile where RegID=@tempRegId

RETURN 0

Git Pull is Not Possible, Unmerged Files

Solved, using the following command set:

git reset --hard
git pull --rebase
git rebase --skip
git pull

The trick is to rebase the changes... We had some trouble rebasing one trivial commit, and so we simply skipped it using git rebase --skip (after having copied the files).

TypeError: 'dict_keys' object does not support indexing

Clearly you're passing in d.keys() to your shuffle function. Probably this was written with python2.x (when d.keys() returned a list). With python3.x, d.keys() returns a dict_keys object which behaves a lot more like a set than a list. As such, it can't be indexed.

The solution is to pass list(d.keys()) (or simply list(d)) to shuffle.

How to have a drop down <select> field in a rails form?

<%= f.select :email_provider, ["gmail","yahoo","msn"]%>

"git checkout <commit id>" is changing branch to "no branch"

If you are branch master and you do a git checkout <SHA>

I'm fairly certain that this causes git to load that commit in a detached state, changing you out of the current branch.

If you want to make changes you can and then you can do a git checkout -b <mynewbranch> to create a new branch based off that commit and any changes you have made.

Get selected option text with JavaScript

Use -

$.trim($("select").children("option:selected").text())   //cat

Here is the fiddle - http://jsfiddle.net/eEGr3/

display Java.util.Date in a specific format

java.time

Here’s the modern answer.

    DateTimeFormatter sourceFormatter = DateTimeFormatter.ofPattern("dd/MM/uuuu");
    DateTimeFormatter displayFormatter = DateTimeFormatter
            .ofLocalizedDate(FormatStyle.SHORT)
            .withLocale(Locale.forLanguageTag("zh-SG"));

    String dateString = "31/05/2011";
    LocalDate date = LocalDate.parse(dateString, sourceFormatter);
    System.out.println(date.format(displayFormatter));

Output from this snippet is:

31/05/11

See if you can live with the 2-digit year. Or use FormatStyle.MEDIUM to obtain 2011?5?31?. I recommend you use Java’s built-in date and time formats when you can. It’s easier and lends itself very well to internationalization.

If you need the exact format you gave, just use the source formatter as display formatter too:

    System.out.println(date.format(sourceFormatter));

31/05/2011

I recommend you don’t use SimpleDateFormat. It’s notoriously troublesome and long outdated. Instead I use java.time, the modern Java date and time API.

To obtain a specific format you need to format the parsed date back into a string. Netiher an old-fashioned Date nor a modern LocalDatecan have a format in it.

Link: Oracle tutorial: Date Time explaining how to use java.time.

Get screen width and height in Android

Kotlin Version via Extension Property

If you want to know the size of the screen in pixels as well as dp, using these extension properties really helps:


DimensionUtils.kt

import android.content.res.Resources
import android.graphics.Rect
import android.graphics.RectF
import android.util.DisplayMetrics
import kotlin.math.roundToInt

/**
 * @author aminography
 */

private val displayMetrics: DisplayMetrics by lazy { Resources.getSystem().displayMetrics }

val screenRectPx: Rect
    get() = displayMetrics.run { Rect(0, 0, widthPixels, heightPixels) }

val screenRectDp: RectF
    get() = displayMetrics.run { RectF(0f, 0f, widthPixels.px2dp, heightPixels.px2dp) }

val Number.px2dp: Float
    get() = this.toFloat() / displayMetrics.density

val Number.dp2px: Int
    get() = (this.toFloat() * displayMetrics.density).roundToInt()


Usage:

class MainActivity : AppCompatActivity() {

    override fun onCreate(savedInstanceState: Bundle?) {
        super.onCreate(savedInstanceState)
        setContentView(R.layout.activity_main)

        val widthPx = screenRectPx.width()
        val heightPx = screenRectPx.height()
        println("[PX] screen width: $widthPx , height: $heightPx")

        val widthDp = screenRectDp.width()
        val heightDp = screenRectDp.height()
        println("[DP] screen width: $widthDp , height: $heightDp")
    }
}

Result:

When the device is in portrait orientation:

[PX] screen width: 1440 , height: 2392
[DP] screen width: 360.0 , height: 598.0

When the device is in landscape orientation:

[PX] screen width: 2392 , height: 1440
[DP] screen width: 598.0 , height: 360.0

Virtualbox "port forward" from Guest to Host

That's not possible. localhost always defaults to the loopback device on the local operating system.
As your virtual machine runs its own operating system it has its own loopback device which you cannot access from the outside.

If you want to access it e.g. in a browser, connect to it using the local IP instead:

http://192.168.180.1:8000

This is just an example of course, you can find out the actual IP by issuing an ifconfig command on a shell in the guest operating system.

How do you copy and paste into Git Bash

For paste in Git BASH CTRL + P

Remove all child nodes from a parent?

A other users suggested,

.empty()

is good enought, because it removes all descendant nodes (both tag-nodes and text-nodes) AND all kind of data stored inside those nodes. See the JQuery's API empty documentation.

If you wish to keep data, like event handlers for example, you should use

.detach()

as described on the JQuery's API detach documentation.

The method .remove() could be usefull for similar purposes.

Escape regex special characters in a Python string

I'm surprised no one has mentioned using regular expressions via re.sub():

import re
print re.sub(r'([\"])',    r'\\\1', 'it\'s "this"')  # it's \"this\"
print re.sub(r"([\'])",    r'\\\1', 'it\'s "this"')  # it\'s "this"
print re.sub(r'([\" \'])', r'\\\1', 'it\'s "this"')  # it\'s\ \"this\"

Important things to note:

  • In the search pattern, include \ as well as the character(s) you're looking for. You're going to be using \ to escape your characters, so you need to escape that as well.
  • Put parentheses around the search pattern, e.g. ([\"]), so that the substitution pattern can use the found character when it adds \ in front of it. (That's what \1 does: uses the value of the first parenthesized group.)
  • The r in front of r'([\"])' means it's a raw string. Raw strings use different rules for escaping backslashes. To write ([\"]) as a plain string, you'd need to double all the backslashes and write '([\\"])'. Raw strings are friendlier when you're writing regular expressions.
  • In the substitution pattern, you need to escape \ to distinguish it from a backslash that precedes a substitution group, e.g. \1, hence r'\\\1'. To write that as a plain string, you'd need '\\\\\\1' — and nobody wants that.

Watch multiple $scope attributes

Angular introduced $watchGroup in version 1.3 using which we can watch multiple variables, with a single $watchGroup block $watchGroup takes array as first parameter in which we can include all of our variables to watch.

$scope.$watchGroup(['var1','var2'],function(newVals,oldVals){
   console.log("new value of var1 = " newVals[0]);
   console.log("new value of var2 = " newVals[1]);
   console.log("old value of var1 = " oldVals[0]);
   console.log("old value of var2 = " oldVals[1]);
});

PHP CURL DELETE request

My own class request with wsse authentication

class Request {

    protected $_url;
    protected $_username;
    protected $_apiKey;

    public function __construct($url, $username, $apiUserKey) {
        $this->_url = $url;     
        $this->_username = $username;
        $this->_apiKey = $apiUserKey;
    }

    public function getHeader() {
        $nonce = uniqid();
        $created = date('c');
        $digest = base64_encode(sha1(base64_decode($nonce) . $created . $this->_apiKey, true));

        $wsseHeader = "Authorization: WSSE profile=\"UsernameToken\"\n";
        $wsseHeader .= sprintf(
            'X-WSSE: UsernameToken Username="%s", PasswordDigest="%s", Nonce="%s", Created="%s"', $this->_username, $digest, $nonce, $created
        );

        return $wsseHeader;
    }

    public function curl_req($path, $verb=NULL, $data=array()) {                    

        $wsseHeader[] = "Accept: application/vnd.api+json";
        $wsseHeader[] = $this->getHeader();

        $options = array(
            CURLOPT_URL => $this->_url . $path,
            CURLOPT_HTTPHEADER => $wsseHeader,
            CURLOPT_RETURNTRANSFER => true, 
            CURLOPT_HEADER => false             
        );                  

        if( !empty($data) ) {
            $options += array(
                CURLOPT_POSTFIELDS => $data,
                CURLOPT_SAFE_UPLOAD => true
            );                          
        }

        if( isset($verb) ) {
            $options += array(CURLOPT_CUSTOMREQUEST => $verb);                          
        }

        $ch = curl_init();
        curl_setopt_array($ch, $options);
        $result = curl_exec($ch);                   

        if(false === $result ) {
            echo curl_error($ch);
        }
        curl_close($ch);

        return $result; 
    }
}

Using async/await with a forEach loop

Bergi's solution works nicely when fs is promise based. You can use bluebird, fs-extra or fs-promise for this.

However, solution for node's native fs libary is as follows:

const result = await Promise.all(filePaths
    .map( async filePath => {
      const fileContents = await getAssetFromCache(filePath, async function() {

        // 1. Wrap with Promise    
        // 2. Return the result of the Promise
        return await new Promise((res, rej) => {
          fs.readFile(filePath, 'utf8', function(err, data) {
            if (data) {
              res(data);
            }
          });
        });
      });

      return fileContents;
    }));

Note: require('fs') compulsorily takes function as 3rd arguments, otherwise throws error:

TypeError [ERR_INVALID_CALLBACK]: Callback must be a function

How to uncheck a radio button?

You can also simulate the radiobutton behavior using only checkboxes:

<input type="checkbox" class="fakeRadio" checked />
<input type="checkbox" class="fakeRadio" />
<input type="checkbox" class="fakeRadio" />

Then, you can use this simple code to work for you:

$(".fakeRadio").click(function(){
    $(".fakeRadio").prop( "checked", false );
    $(this).prop( "checked", true );
});

It works fine and you have more control over the behavior of each button.

You can try it by yourself at: http://jsfiddle.net/almircampos/n1zvrs0c/

This fork also let's you unselect all as requested in a comment: http://jsfiddle.net/5ry8n4f4/

Get program execution time in the shell

one possibly simple method ( that may not meet different users needs ) is the use of shell PROMPT.it is a simple solution that can be useful in some cases. You can use the bash prompting feature as in the example below:

export PS1='[\t \u@\h]\$' 

The above command will result in changing the shell prompt to :

[HH:MM:SS username@hostname]$ 

Each time you run a command (or hit enter) returning back to the shell prompt, the prompt will display current time.

notes:
1) beware that if you waited for sometime before you type your next command, then this time need to be considered, i.e the time displayed in the shell prompt is the timestamp when the shell prompt was displayed, not when you enter command. some users choose to hit Enter key to get a new prompt with a new timestamp before they are ready for the next command.
2) There are other available options and modifiers that can be used to change the bash prompt, refer to ( man bash ) for more details.

Drawing a dot on HTML5 canvas

This should do the job

//get a reference to the canvas
var ctx = $('#canvas')[0].getContext("2d");

//draw a dot
ctx.beginPath();
ctx.arc(20, 20, 10, 0, Math.PI*2, true);
ctx.closePath();
ctx.fill();

Get the Last Inserted Id Using Laravel Eloquent

public function store( UserStoreRequest $request ) {
    $input = $request->all();
    $user = User::create($input);
    $userId=$user->id 
}

How to remove a field completely from a MongoDB document?

To reference a package and remove various "keys", try this

db['name1.name2.name3.Properties'].remove([
{
     "key" : "name_key1"
},
{
     "key" : "name_key2"
},
{
     "key" : "name_key3"
}
)]

Exception : mockito wanted but not invoked, Actually there were zero interactions with this mock

@Jk1's answer is fine, but Mockito also allows for more succinct injection using annotations:

@InjectMocks MyClass myClass; //@InjectMocks automatically instantiates too
@Mock MyInterface myInterface

But regardless of which method you use, the annotations are not being processed (not even your @Mock) unless you somehow call the static MockitoAnnotation.initMocks() or annotate the class with @RunWith(MockitoJUnitRunner.class).

Difference between static STATIC_URL and STATIC_ROOT on Django

All the answers above are helpful but none solved my issue. In my production file, my STATIC_URL was https://<URL>/static and I used the same STATIC_URL in my dev settings.py file.

This causes a silent failure in django/conf/urls/static.py.

The test elif not settings.DEBUG or '://' in prefix: picks up the '//' in the URL and does not add the static URL pattern, causing no static files to be found.

It would be thoughtful if Django spit out an error message stating you can't use a http(s):// with DEBUG = True

I had to change STATIC_URL to be '/static/'

How to get full width in body element

If its in a landscape then you will be needing more width and less height! That's just what all websites have.

Lets go with a basic first then the rest!

The basic CSS:

By CSS you can do this,

#body {
width: 100%;
height: 100%;
}

Here you are using a div with id body, as:

<body>
  <div id="body>
    all the text would go here!
  </div>
</body>

Then you can have a web page with 100% height and width.

What if he tries to resize the window?

The issues pops up, what if he tries to resize the window? Then all the elements inside #body would try to mess up the UI. For that you can write this:

#body {
height: 100%;
width: 100%;
}

And just add min-height max-height min-width and max-width.

This way, the page element would stay at the place they were at the page load.

Using JavaScript:

Using JavaScript, you can control the UI, use jQuery as:

$('#body').css('min-height', '100%');

And all other remaining CSS properties, and JS will take care of the User Interface when the user is trying to resize the window.

How to not add scroll to the web page:

If you are not trying to add a scroll, then you can use this JS

$('#body').css('min-height', screen.height); // or anyother like window.height

This way, the document will get a new height whenever the user would load the page.

Second option is better, because when users would have different screen resolutions they would want a CSS or Style sheet created for their own screen. Not for others!

Tip: So try using JS to find current Screen size and edit the page! :)

Using css transform property in jQuery

$(".oSlider-rotate").slider({
     min: 10,
     max: 74,
     step: .01,
     value: 24,
     slide: function(e,ui){
                 $('.user-text').css('transform', 'scale(' + ui.value + ')')

            }                
  });

This will solve the issue

Where does the iPhone Simulator store its data?

Easiest way ever.

  1. Catch a Breakpoint somewhere.

  2. Enter po NSHomeDirectory() in console window

Result:

(lldb) po NSHomeDirectory() /Users/usernam/Library/Developer/CoreSimulator/Devices/4734F8C7-B90F-4566-8E89-5060505E387F/data/Containers/Data/Application/395818BB-6D0F-499F-AAFE-068A783D9753

POST data to a URL in PHP

Your question is not particularly clear, but in case you want to send POST data to a url without using a form, you can use either fsockopen or curl.

Here's a pretty good walkthrough of both

How to add new contacts in android

There are several good articles on the subject on dev2qa.com site, about Contacts Provider API:

How To Add Contact In Android Programmatically

How To Update Delete Android Contacts Programmatically

How To Get Contact List In Android Programmatically

Contacts are stored in SQLite .db files in bunch of tables, the structure is discussed here: Android Contacts Database Structure

Official Google documentation on Contacts Provider here

Show values from a MySQL database table inside a HTML table on a webpage

OOP Style : At first connection with database.

<?php
class database
{

 public $host = "localhost";
 public $user = "root";
 public $pass = "";
 public $db   = "db";
 public $link;

 public function __construct()
 {
    $this->connect();
 }

 private function connect()
 {
   $this->link = new mysqli($this->host, $this->user, $this->pass, $this->db);
    return $this->link;
 }

 public function select($query)
 {
    $result = $this->link->query($query) or die($this->link->error.__LINE__);

    if($result->num_rows>0)
    {
      return $result;
    } 
    else 
    {
      return false;
    }
}
?>

Then :

    <?php
        $db = new database();

        $query = "select * from data";
        $result = $db->select($query);

        echo "<table>";
         echo "<tr>";
            echo "<th>Name </th>";
            echo "<th>Roll </th>";
         echo "</tr>";
         while($row = mysqli_fetch_array($result)) 
         {
            echo "<tr>";
            echo "<td> $row[name]</td>";
            echo "<td> $row[roll]</td>";
            echo "</tr>";
         }
       echo "</table>";
 ?>

How do I iterate over an NSArray?

The results of the test and source code are below (you can set the number of iterations in the app). The time is in milliseconds, and each entry is an average result of running the test 5-10 times. I found that generally it is accurate to 2-3 significant digits and after that it would vary with each run. That gives a margin of error of less than 1%. The test was running on an iPhone 3G as that's the target platform I was interested in.

numberOfItems   NSArray (ms)    C Array (ms)    Ratio
100             0.39            0.0025          156
191             0.61            0.0028          218
3,256           12.5            0.026           481
4,789           16              0.037           432
6,794           21              0.050           420
10,919          36              0.081           444
19,731          64              0.15            427
22,030          75              0.162           463
32,758          109             0.24            454
77,969          258             0.57            453
100,000         390             0.73            534

The classes provided by Cocoa for handling data sets (NSDictionary, NSArray, NSSet etc.) provide a very nice interface for managing information, without having to worry about the bureaucracy of memory management, reallocation etc. Of course this does come at a cost though. I think it's pretty obvious that say using an NSArray of NSNumbers is going to be slower than a C Array of floats for simple iterations, so I decided to do some tests, and the results were pretty shocking! I wasn't expecting it to be this bad. Note: these tests are conducted on an iPhone 3G as that's the target platform I was interested in.

In this test I do a very simple random access performance comparison between a C float* and NSArray of NSNumbers

I create a simple loop to sum up the contents of each array and time them using mach_absolute_time(). The NSMutableArray takes on average 400 times longer!! (not 400 percent, just 400 times longer! thats 40,000% longer!).

Header:

// Array_Speed_TestViewController.h

// Array Speed Test

// Created by Mehmet Akten on 05/02/2009.

// Copyright MSA Visuals Ltd. 2009. All rights reserved.

#import <UIKit/UIKit.h>

@interface Array_Speed_TestViewController : UIViewController {

    int                     numberOfItems;          // number of items in array

    float                   *cArray;                // normal c array

    NSMutableArray          *nsArray;               // ns array

    double                  machTimerMillisMult;    // multiplier to convert mach_absolute_time() to milliseconds



    IBOutlet    UISlider    *sliderCount;

    IBOutlet    UILabel     *labelCount;


    IBOutlet    UILabel     *labelResults;

}


-(IBAction) doNSArray:(id)sender;

-(IBAction) doCArray:(id)sender;

-(IBAction) sliderChanged:(id)sender;


@end

Implementation:

// Array_Speed_TestViewController.m

// Array Speed Test

// Created by Mehmet Akten on 05/02/2009.

// Copyright MSA Visuals Ltd. 2009. All rights reserved.

    #import "Array_Speed_TestViewController.h"
    #include <mach/mach.h>
    #include <mach/mach_time.h>

 @implementation Array_Speed_TestViewController



 // Implement viewDidLoad to do additional setup after loading the view, typically from a nib.

- (void)viewDidLoad {

    NSLog(@"viewDidLoad");


    [super viewDidLoad];


    cArray      = NULL;

    nsArray     = NULL;


    // read initial slider value setup accordingly

    [self sliderChanged:sliderCount];


    // get mach timer unit size and calculater millisecond factor

    mach_timebase_info_data_t info;

    mach_timebase_info(&info);

    machTimerMillisMult = (double)info.numer / ((double)info.denom * 1000000.0);

    NSLog(@"machTimerMillisMult = %f", machTimerMillisMult);

}



// pass in results of mach_absolute_time()

// this converts to milliseconds and outputs to the label

-(void)displayResult:(uint64_t)duration {

    double millis = duration * machTimerMillisMult;


    NSLog(@"displayResult: %f milliseconds", millis);


    NSString *str = [[NSString alloc] initWithFormat:@"%f milliseconds", millis];

    [labelResults setText:str];

    [str release];

}




// process using NSArray

-(IBAction) doNSArray:(id)sender {

    NSLog(@"doNSArray: %@", sender);


    uint64_t startTime = mach_absolute_time();

    float total = 0;

    for(int i=0; i<numberOfItems; i++) {

        total += [[nsArray objectAtIndex:i] floatValue];

    }

    [self displayResult:mach_absolute_time() - startTime];

}




// process using C Array

-(IBAction) doCArray:(id)sender {

    NSLog(@"doCArray: %@", sender);


    uint64_t start = mach_absolute_time();

    float total = 0;

    for(int i=0; i<numberOfItems; i++) {

        total += cArray[i];

    }

    [self displayResult:mach_absolute_time() - start];

}



// allocate NSArray and C Array 

-(void) allocateArrays {

    NSLog(@"allocateArrays");


    // allocate c array

    if(cArray) delete cArray;

    cArray = new float[numberOfItems];


    // allocate NSArray

    [nsArray release];

    nsArray = [[NSMutableArray alloc] initWithCapacity:numberOfItems];



    // fill with random values

    for(int i=0; i<numberOfItems; i++) {

        // add number to c array

        cArray[i] = random() * 1.0f/(RAND_MAX+1);


        // add number to NSArray

        NSNumber *number = [[NSNumber alloc] initWithFloat:cArray[i]];

        [nsArray addObject:number];

        [number release];

    }


}



// callback for when slider is changed

-(IBAction) sliderChanged:(id)sender {

    numberOfItems = sliderCount.value;

    NSLog(@"sliderChanged: %@, %i", sender, numberOfItems);


    NSString *str = [[NSString alloc] initWithFormat:@"%i items", numberOfItems];

    [labelCount setText:str];

    [str release];


    [self allocateArrays];

}



//cleanup

- (void)dealloc {

    [nsArray release];

    if(cArray) delete cArray;


    [super dealloc];

}


@end

From : memo.tv

////////////////////

Available since the introduction of blocks, this allows to iterate an array with blocks. Its syntax isn't as nice as fast enumeration, but there is one very interesting feature: concurrent enumeration. If enumeration order is not important and the jobs can be done in parallel without locking, this can provide a considerable speedup on a multi-core system. More about that in the concurrent enumeration section.

[myArray enumerateObjectsUsingBlock:^(id object, NSUInteger index, BOOL *stop) {
    [self doSomethingWith:object];
}];
[myArray enumerateObjectsWithOptions:NSEnumerationConcurrent usingBlock:^(id obj, NSUInteger idx, BOOL *stop) {
    [self doSomethingWith:object];
}];

/////////// NSFastEnumerator

The idea behind fast enumeration is to use fast C array access to optimize iteration. Not only is it supposed to be faster than traditional NSEnumerator, but Objective-C 2.0 also provides a very concise syntax.

id object;
for (object in myArray) {
    [self doSomethingWith:object];
}

/////////////////

NSEnumerator

This is a form of external iteration: [myArray objectEnumerator] returns an object. This object has a method nextObject that we can call in a loop until it returns nil

NSEnumerator *enumerator = [myArray objectEnumerator];
id object;
while (object = [enumerator nextObject]) {
    [self doSomethingWith:object];
}

/////////////////

objectAtIndex: enumeration

Using a for loop which increases an integer and querying the object using [myArray objectAtIndex:index] is the most basic form of enumeration.

NSUInteger count = [myArray count];
for (NSUInteger index = 0; index < count ; index++) {
    [self doSomethingWith:[myArray objectAtIndex:index]];
}

////////////// From : darkdust.net

How to convert JSON to XML or XML to JSON?

For convert JSON string to XML try this:

    public string JsonToXML(string json)
    {
        XDocument xmlDoc = new XDocument(new XDeclaration("1.0", "utf-8", ""));
        XElement root = new XElement("Root");
        root.Name = "Result";

        var dataTable = JsonConvert.DeserializeObject<DataTable>(json);
        root.Add(
                 from row in dataTable.AsEnumerable()
                 select new XElement("Record",
                                     from column in dataTable.Columns.Cast<DataColumn>()
                                     select new XElement(column.ColumnName, row[column])
                                    )
               );


        xmlDoc.Add(root);
        return xmlDoc.ToString();
    }

For convert XML to JSON try this:

    public string XmlToJson(string xml)
    {
       XmlDocument doc = new XmlDocument();
       doc.LoadXml(xml);

       string jsonText = JsonConvert.SerializeXmlNode(doc);
       return jsonText;
     }

Get POST data in C#/ASP.NET

I'm a little surprised that this question has been asked so many times before, but the most reuseable and friendly solution hasn't been documented.

I often have webpages using AngularJS, and when I click on a Save button, I'll "POST" this data back to my .aspx page or .ashx handler to save this back to the database. The data will be in the form of a JSON record.

On the server, to turn the raw posted data back into a C# class, here's what I would do.

First, define a C# class which will contain the posted data.

Supposing my webpage is posting JSON data like this:

{
    "UserID" : 1,
    "FirstName" : "Mike",
    "LastName" : "Mike",
    "Address1" : "10 Really Street",
    "Address2" : "London"
}

Then I'd define a C# class like this...

public class JSONRequest
{
    public int UserID { get; set; }
    public string FirstName { get; set; }
    public string LastName { get; set; }
    public string Address1 { get; set; }
    public string Address2 { get; set; }
}

(These classes can be nested, but the structure must match the format of the JSON data. So, if you're posting a JSON User record, with a list of Order records within it, your C# class should also contain a List<> of Order records.)

Now, in my .aspx.cs or .ashx file, I just need to do this, and leave JSON.Net to do the hard work...

    protected void Page_Load(object sender, EventArgs e)
    {
        string jsonString = "";
        HttpContext.Current.Request.InputStream.Position = 0;
        using (StreamReader inputStream = new StreamReader(this.Request.InputStream))
        {
            jsonString = inputStream.ReadToEnd();
        }
        JSONRequest oneQuestion = JsonConvert.DeserializeObject<JSONRequest>(jsonString);

And that's it. You now have a JSONRequest class containing the various fields which were POSTed to your server.

Where to find htdocs in XAMPP Mac

Make sure no other apache servers are running as it generates an error when you try to access it on the browser even with a different port. Go to Finder and below Device you will usually see the lampp icon. You can also open the htdocs from any of the ide or code editor by opening files or project once you locate the lampp icon. Make sure you mount the stack.

How do you attach and detach from Docker's process?

To detach from a running container, use ^P^Q (hold Ctrl, press P, press Q, release Ctrl).

There's a catch: this only works if the container was started with both -t and -i.

If you have a running container that was started without one (or both) of these options, and you attach with docker attach, you'll need to find another way to detach. Depending on the options you chose and the program that's running, ^C may work, or it may kill the whole container. You'll have to experiment.

Another catch: Depending on the programs you're using, your terminal, shell, SSH client, or multiplexer could be intercepting either ^P or ^Q (usually the latter). To test whether this is the issue, try running or attaching with the --detach-keys z argument. You should now be able to detach by pressing z, without any modifiers. If this works, another program is interfering. The easiest way to work around this is to set your own detach sequence using the --detach-keys argument. (For example, to exit with ^K, use --detach-keys 'ctrl-k'.) Alternatively, you can attempt to disable interception of the keys in your terminal or other interfering program. For example, stty start '' or stty start undef may prevent the terminal from intercepting ^Q on some POSIX systems, though I haven't found this to be helpful.

Oracle - How to create a materialized view with FAST REFRESH and JOINS

The key checks for FAST REFRESH includes the following:

1) An Oracle materialized view log must be present for each base table.
2) The RowIDs of all the base tables must appear in the SELECT list of the MVIEW query definition.
3) If there are outer joins, unique constraints must be placed on the join columns of the inner table.

No 3 is easy to miss and worth highlighting here

How to set some xlim and ylim in Seaborn lmplot facetgrid

The lmplot function returns a FacetGrid instance. This object has a method called set, to which you can pass key=value pairs and they will be set on each Axes object in the grid.

Secondly, you can set only one side of an Axes limit in matplotlib by passing None for the value you want to remain as the default.

Putting these together, we have:

g = sns.lmplot('X', 'Y', df, col='Z', sharex=False, sharey=False)
g.set(ylim=(0, None))

enter image description here

What is wrong with this code that uses the mysql extension to fetch data from a database in PHP?

Try

$query = mysql_query("SELECT * FROM users WHERE name = 'Admin' ")or die(mysql_error());

and check if this throw any error.

Then use while($rows = mysql_fetch_assoc($query)):

And finally display it as

echo $name . "<br/>" . $address . "<br/>" . $email . "<br/>" . $subject . "<br/>" . $comment . "<br/><br/>" . ;

Do not user mysql_* as its deprecated.

How do I move files in node.js?

According to seppo0010 comment, I used the rename function to do that.

http://nodejs.org/docs/latest/api/fs.html#fs_fs_rename_oldpath_newpath_callback

fs.rename(oldPath, newPath, callback)

Added in: v0.0.2

oldPath <String> | <Buffer>
newPath <String> | <Buffer>
callback <Function>

Asynchronous rename(2). No arguments other than a possible exception are given to the completion callback.

AngularJS : Custom filters and ng-repeat

If you still want a custom filter you can pass in the search model to the filter:

<article data-ng-repeat="result in results | cartypefilter:search" class="result">

Where definition for the cartypefilter can look like this:

app.filter('cartypefilter', function() {
  return function(items, search) {
    if (!search) {
      return items;
    }

    var carType = search.carType;
    if (!carType || '' === carType) {
      return items;
    }

    return items.filter(function(element, index, array) {
      return element.carType.name === search.carType;
    });

  };
});

http://plnkr.co/edit/kBcUIayO8tQsTTjTA2vO?p=preview

java.io.FileNotFoundException: the system cannot find the file specified

I was reading path from a properties file and didn't mention there was a space in the end. Make sure you don't have one.

How to import an existing X.509 certificate and private key in Java keystore to use in SSL?

And one more:

#!/bin/bash

# We have:
#
# 1) $KEY : Secret key in PEM format ("-----BEGIN RSA PRIVATE KEY-----") 
# 2) $LEAFCERT : Certificate for secret key obtained from some
#    certification outfit, also in PEM format ("-----BEGIN CERTIFICATE-----")   
# 3) $CHAINCERT : Intermediate certificate linking $LEAFCERT to a trusted
#    Self-Signed Root CA Certificate 
#
# We want to create a fresh Java "keystore" $TARGET_KEYSTORE with the
# password $TARGET_STOREPW, to be used by Tomcat for HTTPS Connector.
#
# The keystore must contain: $KEY, $LEAFCERT, $CHAINCERT
# The Self-Signed Root CA Certificate is obtained by Tomcat from the
# JDK's truststore in /etc/pki/java/cacerts

# The non-APR HTTPS connector (APR uses OpenSSL-like configuration, much
# easier than this) in server.xml looks like this 
# (See: https://tomcat.apache.org/tomcat-6.0-doc/ssl-howto.html):
#
#  <Connector port="8443" protocol="org.apache.coyote.http11.Http11Protocol"
#                SSLEnabled="true"
#                maxThreads="150" scheme="https" secure="true"
#                clientAuth="false" sslProtocol="TLS"
#                keystoreFile="/etc/tomcat6/etl-web.keystore.jks"
#                keystorePass="changeit" />
#

# Let's roll:    

TARGET_KEYSTORE=/etc/tomcat6/foo-server.keystore.jks
TARGET_STOREPW=changeit

TLS=/etc/pki/tls

KEY=$TLS/private/httpd/foo-server.example.com.key
LEAFCERT=$TLS/certs/httpd/foo-server.example.com.pem
CHAINCERT=$TLS/certs/httpd/chain.cert.pem

# ----
# Create PKCS#12 file to import using keytool later
# ----

# From https://www.sslshopper.com/ssl-converter.html:
# The PKCS#12 or PFX format is a binary format for storing the server certificate,
# any intermediate certificates, and the private key in one encryptable file. PFX
# files usually have extensions such as .pfx and .p12. PFX files are typically used 
# on Windows machines to import and export certificates and private keys.

TMPPW=$$ # Some random password

PKCS12FILE=`mktemp`

if [[ $? != 0 ]]; then
  echo "Creation of temporary PKCS12 file failed -- exiting" >&2; exit 1
fi

TRANSITFILE=`mktemp`

if [[ $? != 0 ]]; then
  echo "Creation of temporary transit file failed -- exiting" >&2; exit 1
fi

cat "$KEY" "$LEAFCERT" > "$TRANSITFILE"

openssl pkcs12 -export -passout "pass:$TMPPW" -in "$TRANSITFILE" -name etl-web > "$PKCS12FILE"

/bin/rm "$TRANSITFILE"

# Print out result for fun! Bug in doc (I think): "-pass " arg does not work, need "-passin"

openssl pkcs12 -passin "pass:$TMPPW" -passout "pass:$TMPPW" -in "$PKCS12FILE" -info

# ----
# Import contents of PKCS12FILE into a Java keystore. WTF, Sun, what were you thinking?
# ----

if [[ -f "$TARGET_KEYSTORE" ]]; then
  /bin/rm "$TARGET_KEYSTORE"
fi

keytool -importkeystore \
   -deststorepass  "$TARGET_STOREPW" \
   -destkeypass    "$TARGET_STOREPW" \
   -destkeystore   "$TARGET_KEYSTORE" \
   -srckeystore    "$PKCS12FILE" \
   -srcstoretype  PKCS12 \
   -srcstorepass  "$TMPPW" \
   -alias foo-the-server

/bin/rm "$PKCS12FILE"

# ----
# Import the chain certificate. This works empirically, it is not at all clear from the doc whether this is correct
# ----

echo "Importing chain"

TT=-trustcacerts

keytool -import $TT -storepass "$TARGET_STOREPW" -file "$CHAINCERT" -keystore "$TARGET_KEYSTORE" -alias chain

# ----
# Print contents
# ----

echo "Listing result"

keytool -list -storepass "$TARGET_STOREPW" -keystore "$TARGET_KEYSTORE"

How can I clear the terminal in Visual Studio Code?

I am using Visual Studio Code 1.52.1 on windows 10 machine.'cls' or 'Clear' doesn't clear the terminal.

just write

exit

It will close the terminal and press

ctrl+shift+`

to open new terminal.

Can't specify the 'async' modifier on the 'Main' method of a console app

C# 7.1 (using vs 2017 update 3) introduces async main

You can write:

   static async Task Main(string[] args)
  {
    await ...
  }

For more details C# 7 Series, Part 2: Async Main

Update:

You may get a compilation error:

Program does not contain a static 'Main' method suitable for an entry point

This error is due to that vs2017.3 is configured by default as c#7.0 not c#7.1.

You should explicitly modify the setting of your project to set c#7.1 features.

You can set c#7.1 by two methods:

Method 1: Using the project settings window:

  • Open the settings of your project
  • Select the Build tab
  • Click the Advanced button
  • Select the version you want As shown in the following figure:

enter image description here

Method2: Modify PropertyGroup of .csproj manually

Add this property:

    <LangVersion>7.1</LangVersion>

example:

    <PropertyGroup Condition=" '$(Configuration)|$(Platform)' == 'Debug|AnyCPU' ">
        <PlatformTarget>AnyCPU</PlatformTarget>
        <DebugSymbols>true</DebugSymbols>
        <DebugType>full</DebugType>
        <Optimize>false</Optimize>
        <OutputPath>bin\Debug\</OutputPath>
        <DefineConstants>DEBUG;TRACE</DefineConstants>
        <ErrorReport>prompt</ErrorReport>
        <WarningLevel>4</WarningLevel>
        <Prefer32Bit>false</Prefer32Bit>
        <LangVersion>7.1</LangVersion>
    </PropertyGroup>    

Error "can't load package: package my_prog: found packages my_prog and main"

Make sure that your package is installed in your $GOPATH directory or already inside your workspace/package.

For example: if your $GOPATH = "c:\go", make sure that the package inside C:\Go\src\pkgName

Multiple radio button groups in one form

Set equal name attributes to create a group;

_x000D_
_x000D_
<form>_x000D_
  <fieldset id="group1">_x000D_
    <input type="radio" value="value1" name="group1">_x000D_
    <input type="radio" value="value2" name="group1">_x000D_
  </fieldset>_x000D_
_x000D_
  <fieldset id="group2">_x000D_
    <input type="radio" value="value1" name="group2">_x000D_
    <input type="radio" value="value2" name="group2">_x000D_
    <input type="radio" value="value3" name="group2">_x000D_
  </fieldset>_x000D_
</form>
_x000D_
_x000D_
_x000D_

Bootstrap footer at the bottom of the page

Use this stylesheet:

_x000D_
_x000D_
/* Sticky footer styles_x000D_
-------------------------------------------------- */_x000D_
html {_x000D_
  position: relative;_x000D_
  min-height: 100%;_x000D_
}_x000D_
body {_x000D_
  /* Margin bottom by footer height */_x000D_
  margin-bottom: 60px;_x000D_
}_x000D_
.footer {_x000D_
  position: absolute;_x000D_
  bottom: 0;_x000D_
  width: 100%;_x000D_
  /* Set the fixed height of the footer here */_x000D_
  height: 60px;_x000D_
  line-height: 60px; /* Vertically center the text there */_x000D_
  background-color: #f5f5f5;_x000D_
}_x000D_
_x000D_
_x000D_
/* Custom page CSS_x000D_
-------------------------------------------------- */_x000D_
/* Not required for template or sticky footer method. */_x000D_
_x000D_
body > .container {_x000D_
  padding: 60px 15px 0;_x000D_
}_x000D_
_x000D_
.footer > .container {_x000D_
  padding-right: 15px;_x000D_
  padding-left: 15px;_x000D_
}_x000D_
_x000D_
code {_x000D_
  font-size: 80%;_x000D_
}
_x000D_
_x000D_
_x000D_

This declaration has no storage class or type specifier in C++

Calling m.check(side), meaning you are running actual code, but you can't run code outside main() - you can only define variables. In C++, code can only appear inside function bodies or in variable initializes.

How to open local files in Swagger-UI

In a local directory that contains the file ./docs/specs/openapi.yml that you want to view, you can run the following to start a container and access the spec at http://127.0.0.1:8246.

docker run -t -i -p 8246:8080 -e SWAGGER_JSON=/var/specs/openapi.yml -v $PWD/docs/specs:/var/specs swaggerapi/swagger-ui

How to create checkbox inside dropdown?

You can always use multiple or multiple = "true" option with a select tag, but there is one jquery plugin which makes it more beautiful. It is called chosen and can be found here.

This fiddle-example might help you to get started

Thank you.

Slice indices must be integers or None or have __index__ method

Your debut and fin values are floating point values, not integers, because taille is a float.

Make those values integers instead:

item = plateau[int(debut):int(fin)]

Alternatively, make taille an integer:

taille = int(sqrt(len(plateau)))

Is it possible to put a ConstraintLayout inside a ScrollView?

Anyone who has set below property to

ScrollView:: android:fillViewport="true"

constraint layout: android:layout_height="wrap_content"

And it's still not working then make sure then you have not set the Inner scrollable layout (RecycleView) bottom constraint to bottom of the parent.

Add below lines of code:

android:nestedScrollingEnabled="false"
android:layout_height="wrap_content"

Make sure to remove below constraint:

app:layout_constraintBottom_toBottomOf="parent"

Full code

   <androidx.core.widget.NestedScrollView
    android:layout_width="match_parent"
    android:layout_height="match_parent"
    android:fillViewport="true">

<androidx.constraintlayout.widget.ConstraintLayout
    android:id="@+id/selectHubLayout"
    android:layout_width="match_parent"
    android:layout_height="wrap_content"
    tools:context=".ui.hubs.SelectHubFragment">

    <include
        android:id="@+id/include"
        layout="@layout/signup_hub_selection_details"
        app:layout_constraintEnd_toEndOf="parent"
        app:layout_constraintStart_toStartOf="parent"
        app:layout_constraintTop_toTopOf="parent" />

    <androidx.recyclerview.widget.RecyclerView
        android:id="@+id/rv_HubSelection"
        android:layout_width="0dp"
        android:layout_height="wrap_content"
        android:layout_marginTop="8dp"
        android:nestedScrollingEnabled="false"
        app:layoutManager="androidx.recyclerview.widget.LinearLayoutManager"
        app:layout_constraintEnd_toEndOf="parent"
        app:layout_constraintHorizontal_bias="1.0"
        app:layout_constraintStart_toStartOf="parent"
        app:layout_constraintTop_toBottomOf="@+id/include" />
</androidx.constraintlayout.widget.ConstraintLayout>

How to Animate Addition or Removal of Android ListView Rows

After inserting new row to ListView, I just scroll the ListView to new position.

ListView.smoothScrollToPosition(position);

How do I Validate the File Type of a File Upload?

As some people have mentioned, Javascript is the way to go. Bear in mind that the "validation" here is only by file extension, it won't validate that the file is a real excel spreadsheet!

TypeError: 'undefined' is not an object

try out this if you want to assign value to object and it is showing this error in angular..

crate object in construtor

this.modelObj = new Model(); //<---------- after declaring object above

JQuery ajax call default timeout value

The XMLHttpRequest.timeout property represents a number of milliseconds a request can take before automatically being terminated. The default value is 0, which means there is no timeout. An important note the timeout shouldn't be used for synchronous XMLHttpRequests requests, used in a document environment or it will throw an InvalidAccessError exception. You may not use a timeout for synchronous requests with an owning window.

IE10 and 11 do not support synchronous requests, with support being phased out in other browsers too. This is due to detrimental effects resulting from making them.

More info can be found here.

HTML5 best practices; section/header/aside/article elements

The main mistake: You have "divitis" in the whole document.
Why this?

<header>
    <div id="logo"></div>
    <div id="language"></div>
</header>

Instead of:

<header role="banner">
    <!-- Not the best -->
    <h1 id="logo"></h1> <!-- or <figure> and <figcaption>, or <h1> and <p>... -->
    <h2 id="language"></h2>

    <!-- Better, if the IDs have not semantic sense -->
    <h1></h1>
    <h2></h2>
</header>

To stylize this header, use CSS hierarchy: body > section > header > h1, body > section > header > h2

More, ...line 63: why header wraps h2?? If you do not include any more element inside header, just use a single h2.
Keep in mind, your structure is not to stylize document, but construct a document self-explained.

Apply this to the rest of document; Good luck ;)

How to convert strings into integers in Python?

In Python 3.5.1 things like these work:

c = input('Enter number:')
print (int(float(c)))
print (round(float(c)))

and

Enter number:  4.7
4
5

George.

#1062 - Duplicate entry for key 'PRIMARY'

I solved it by changing the "lock" property from "shared" to "exclusive":

ALTER TABLE `table` 
CHANGE COLUMN `ID` `ID` INT(11) NOT NULL AUTO_INCREMENT COMMENT '' , LOCK = EXCLUSIVE;

How do you add a Dictionary of items into another Dictionary

You can try this

var dict1 = ["a" : "foo"]
var dict2 = ["b" : "bar"]

var temp = NSMutableDictionary(dictionary: dict1);
temp.addEntriesFromDictionary(dict2)

Can .NET load and parse a properties file equivalent to Java Properties class?

C# generally uses xml-based config files rather than the *.ini-style file like you said, so there's nothing built-in to handle this. However, google returns a number of promising results.

How to retrieve the first word of the output of a command in bash?

echo "word1 word2" | cut -f 1 -d " "

cut cuts the 1st field (-f 1) from a list of fields delimited by the string " " (-d " ")

How to check object is nil or not in swift?

Swift 4 You cannot compare Any to nil.Because an optional can be nil and hence it always succeeds to true. The only way is to cast it to your desired object and compare it to nil.

if (someone as? String) != nil
{
   //your code`enter code here`
}

Must JDBC Resultsets and Statements be closed separately although the Connection is closed afterwards?

I'm now using Oracle with Java. Here my point of view :

You should close ResultSet and Statement explicitly because Oracle has problems previously with keeping the cursors open even after closing the connection. If you don't close the ResultSet (cursor) it will throw an error like Maximum open cursors exceeded.

I think you may encounter with the same problem with other databases you use.

Here is tutorial Close ResultSet when finished:

Close ResultSet when finished

Close ResultSet object as soon as you finish working with ResultSet object even though Statement object closes the ResultSet object implicitly when it closes, closing ResultSet explicitly gives chance to garbage collector to recollect memory as early as possible because ResultSet object may occupy lot of memory depending on query.

ResultSet.close();

Two-way SSL clarification

In two way ssl the client asks for servers digital certificate and server ask for the same from the client. It is more secured as it is both ways, although its bit slow. Generally we dont follow it as the server doesnt care about the identity of the client, but a client needs to make sure about the integrity of server it is connecting to.

calculate the mean for each column of a matrix in R

Another way is to use purrr package

# example data like what is said above

@A Handcart And Mohair

set.seed(1)
m <- data.frame(matrix(sample(100, 20, replace = TRUE), ncol = 4))


library(purrr)
means <- map_dbl(m, mean)

> means
#  X1   X2   X3   X4 
#47.0 64.4 44.8 67.8 

How do I launch the Android emulator from the command line?

See below instructions for Ubuntu Linux with zsh:

  1. Open terminal window (CTRL+ALT+T)
  2. Run command nano ~/.zshrc to edit your profile
  3. Add following lines in the opened file:
export ANDROID_SDK_HOME="~/Android/Sdk"
alias emulator="$ANDROID_SDK_HOME/emulator/emulator"
  1. Save the file (CTRL+O, CTRL+X)
  2. Source the profile by running command source ~/.zshrc or just log out and log back in
  3. Test by running the command emulator -help in terminal

NOTE: Should be same for bash by replacing .zshrc with .bashrc

How do I increase the capacity of the Eclipse output console?

Window > Preferences, go to the Run/Debug > Console section >> "Limit console output.>>Console buffer size(characters):" (This option can be seen in Eclipse Indigo ,but it limits buffer size at 1,000,000 )

How to consume a SOAP web service in Java

There are many options to consume a SOAP web service with Stub or Java classes created based on WSDL. But if anyone wants to do this without any Java class created, this article is very helpful. Code Snippet from the article:

public String someMethod() throws MalformedURLException, IOException {

//Code to make a webservice HTTP request
String responseString = "";
String outputString = "";
String wsURL = "<Endpoint of the webservice to be consumed>";
URL url = new URL(wsURL);
URLConnection connection = url.openConnection();
HttpURLConnection httpConn = (HttpURLConnection)connection;
ByteArrayOutputStream bout = new ByteArrayOutputStream();
String xmlInput = "entire SOAP Request";

byte[] buffer = new byte[xmlInput.length()];
buffer = xmlInput.getBytes();
bout.write(buffer);
byte[] b = bout.toByteArray();
String SOAPAction = "<SOAP action of the webservice to be consumed>";
// Set the appropriate HTTP parameters.
httpConn.setRequestProperty("Content-Length",
String.valueOf(b.length));
httpConn.setRequestProperty("Content-Type", "text/xml; charset=utf-8");
httpConn.setRequestProperty("SOAPAction", SOAPAction);
httpConn.setRequestMethod("POST");
httpConn.setDoOutput(true);
httpConn.setDoInput(true);
OutputStream out = httpConn.getOutputStream();
//Write the content of the request to the outputstream of the HTTP Connection.
out.write(b);
out.close();
//Ready with sending the request.

//Read the response.
InputStreamReader isr = null;
if (httpConn.getResponseCode() == 200) {
  isr = new InputStreamReader(httpConn.getInputStream());
} else {
  isr = new InputStreamReader(httpConn.getErrorStream());
}

BufferedReader in = new BufferedReader(isr);

//Write the SOAP message response to a String.
while ((responseString = in.readLine()) != null) {
outputString = outputString + responseString;
}
//Parse the String output to a org.w3c.dom.Document and be able to reach every node with the org.w3c.dom API.
Document document = parseXmlFile(outputString); // Write a separate method to parse the xml input.
NodeList nodeLst = document.getElementsByTagName("<TagName of the element to be retrieved>");
String elementValue = nodeLst.item(0).getTextContent();
System.out.println(elementValue);

//Write the SOAP message formatted to the console.
String formattedSOAPResponse = formatXML(outputString); // Write a separate method to format the XML input.
System.out.println(formattedSOAPResponse);
return elementValue;
}

For those who're looking for a similar kind of solution with file upload while consuming a SOAP API, please refer to this post: How to attach a file (pdf, jpg, etc) in a SOAP POST request?

calling a java servlet from javascript

I really recommend you use jquery for the javascript calls and some implementation of JSR311 like jersey for the service layer, which would delegate to your controllers.

This will help you with all the underlying logic of handling the HTTP calls and your data serialization, which is a big help.

PHP - Check if two arrays are equal

Compare them as other values:

if($array_a == $array_b) {
  //they are the same
}

You can read about all array operators here: http://php.net/manual/en/language.operators.array.php Note for example that === also checks that the types and order of the elements in the arrays are the same.

CSS3 selector :first-of-type with class name?

I found a solution for your reference. from some group divs select from group of two same class divs the first one

p[class*="myclass"]:not(:last-of-type) {color:red}
p[class*="myclass"]:last-of-type {color:green}

BTW, I don't know why :last-of-type works, but :first-of-type does not work.

My experiments on jsfiddle... https://jsfiddle.net/aspanoz/m1sg4496/

Create patch or diff file from git repository and apply it to another different git repository

As a complementary, to produce patch for only one specific commit, use:

git format-patch -1 <sha>

When the patch file is generated, make sure your other repo knows where it is when you use git am ${patch-name}

Before adding the patch, use git apply --check ${patch-name} to make sure that there is no confict.

Nullable property to entity field, Entity Framework through Code First

Jon's answer didn't work for me as I got a compiler error CS0453 C# The type must be a non-nullable value type in order to use it as parameter 'T' in the generic type or method

This worked for me though:

protected override void OnModelCreating(DbModelBuilder modelBuilder)
{
    modelBuilder.Entity<SomeObject>().HasOptional(m => m.somefield);
    base.OnModelCreating(modelBuilder);
}

Find all matches in workbook using Excel VBA

You may use the Range.Find method:

http://msdn.microsoft.com/en-us/library/office/ff839746.aspx

This will get you the first cell which contains the search string. By repeating this with setting the "After" argument to the next cell you will get all other occurrences until you are back at the first occurrence.

This will likely be much faster.

Installing a dependency with Bower from URL and specify version

I believe that specifying version works only for git-endpoints. And not for folder/zip ones. As when you point bower to a js-file/folder/zip you already specified package and version (except for js indeed). Because a package has bower.json with version in it. Specifying a version in 'bower install' makes sense when you're pointing bower to a repository which can have many versions of a package. It can be only git I think.

Python NameError: name is not defined

You must define the class before creating an instance of the class. Move the invocation of Something to the end of the script.

You can try to put the cart before the horse and invoke procedures before they are defined, but it will be an ugly hack and you will have to roll your own as defined here:

Make function definition in a python file order independent

How to do a non-greedy match in grep?

The short answer is using the next regular expression:

(?s)<car .*? model=BMW .*?>.*?</car>
  • (?s) - this makes a match across multiline
  • .*? - matches any character, a number of times in a lazy way (minimal match)

A (little) more complicated answer is:

(?s)<([a-z\-_0-9]+?) .*? model=BMW .*?>.*?</\1>

This will makes possible to match car1 and car2 in the following text

<car1 ... model=BMW ...>
...
...
...
</car1>
<car2 ... model=BMW ...>
...
...
...
</car2>
  • (..) represents a capturing group
  • \1 in this context matches the sametext as most recently matched by capturing group number 1

Constructors in Go

I am new to go. I have another pattern taken from other languages, that have constructors. And will work in go.

  1. Create an init method.
  2. Make the init method an (object) once routine. It only runs the first time it is called (per object).
func (d *my_struct) Init (){
    //once
    if !d.is_inited {
        d.is_inited = true
        d.value1 = 7
        d.value2 = 6
    }
}
  1. Call init at the top of every method of this class.

This pattern is also useful, when you need late initialisation (constructor is too early).

Advantages: it hides all the complexity in the class, clients don't need to do anything.

Disadvantages: you must remember to call Init at the top of every method of the class.

Convert UTC date time to local date time

@Adorojan's answer is almost correct. But addition of offset is not correct since offset value will be negative if browser date is ahead of GMT and vice versa. Below is the solution which I came with and is working perfectly fine for me:

_x000D_
_x000D_
// Input time in UTC_x000D_
var inputInUtc = "6/29/2011 4:52:48";_x000D_
_x000D_
var dateInUtc = new Date(Date.parse(inputInUtc+" UTC"));_x000D_
//Print date in UTC time_x000D_
document.write("Date in UTC : " + dateInUtc.toISOString()+"<br>");_x000D_
_x000D_
var dateInLocalTz = convertUtcToLocalTz(dateInUtc);_x000D_
//Print date in local time_x000D_
document.write("Date in Local : " + dateInLocalTz.toISOString());_x000D_
_x000D_
function convertUtcToLocalTz(dateInUtc) {_x000D_
  //Convert to local timezone_x000D_
  return new Date(dateInUtc.getTime() - dateInUtc.getTimezoneOffset()*60*1000);_x000D_
}
_x000D_
_x000D_
_x000D_

Mock MVC - Add Request Parameter to test

When i analyzed your code. I have also faced the same problem but my problem is if i give value for both first and last name means it is working fine. but when i give only one value means it says 400. anyway use the .andDo(print()) method to find out the error

public void testGetUserByName() throws Exception {
    String firstName = "Jack";
    String lastName = "s";       
    this.userClientObject = client.createClient();
    mockMvc.perform(get("/byName")
            .sessionAttr("userClientObject", this.userClientObject)
            .param("firstName", firstName)
            .param("lastName", lastName)               
    ).andDo(print())
     .andExpect(status().isOk())
            .andExpect(content().contentType("application/json"))
            .andExpect(jsonPath("$[0].id").exists())
            .andExpect(jsonPath("$[0].fn").value("Marge"));
}

If your problem is org.springframework.web.bind.missingservletrequestparameterexception you have to change your code to

@RequestMapping(value = "/byName", method = RequestMethod.GET)
    @ResponseStatus(HttpStatus.OK)
    public
    @ResponseBody
    String getUserByName(
        @RequestParam( value="firstName",required = false) String firstName,
        @RequestParam(value="lastName",required = false) String lastName, 
        @ModelAttribute("userClientObject") UserClient userClient)
    {

        return client.getUserByName(userClient, firstName, lastName);
    }

Auto-size dynamic text to fill fixed size container

Most of the other answers use a loop to reduce the font-size until it fits on the div, this is VERY slow since the page needs to re-render the element each time the font changes size. I eventually had to write my own algorithm to make it perform in a way that allowed me to update its contents periodically without freezing the user browser. I added some other functionality (rotating text, adding padding) and packaged it as a jQuery plugin, you can get it at:

https://github.com/DanielHoffmann/jquery-bigtext

simply call

$("#text").bigText();

and it will fit nicely on your container.

See it in action here:

http://danielhoffmann.github.io/jquery-bigtext/

For now it has some limitations, the div must have a fixed height and width and it does not support wrapping text into multiple lines.

I will work on getting an option to set the maximum font-size.

Edit: I have found some more problems with the plugin, it does not handle other box-model besides the standard one and the div can't have margins or borders. I will work on it.

Edit2: I have now fixed those problems and limitations and added more options. You can set maximum font-size and you can also choose to limit the font-size using either width, height or both. I will work into accepting a max-width and max-height values in the wrapper element.

Edit3: I have updated the plugin to version 1.2.0. Major cleanup on the code and new options (verticalAlign, horizontalAlign, textAlign) and support for inner elements inside the span tag (like line-breaks or font-awesome icons.)

How to select the Date Picker In Selenium WebDriver

DatePicker are not Select element. What your doing in your code is wrong.

Datepicker are in fact table with set of rows and columns.To select a date you just have to navigate to the cell where our desired date is present.

So your code should be like this:

WebElement dateWidget = driver.findElement(your locator);
List<WebElement> columns=dateWidget.findElements(By.tagName("td"));

for (WebElement cell: columns){
   //Select 13th Date 
   if (cell.getText().equals("13")){
      cell.findElement(By.linkText("13")).click();
      break;
 }

How can I read a whole file into a string variable

Use ioutil.ReadFile:

func ReadFile(filename string) ([]byte, error)

ReadFile reads the file named by filename and returns the contents. A successful call returns err == nil, not err == EOF. Because ReadFile reads the whole file, it does not treat an EOF from Read as an error to be reported.

You will get a []byte instead of a string. It can be converted if really necessary:

s := string(buf)

Removing X-Powered-By

If you use FastCGI try:

fastcgi_hide_header X-Powered-By;

How to scroll to top of a div using jQuery?

Here is what you can do using jquery:

$('#A_ID').click(function (e) { //#A_ID is an example. Use the id of your Anchor
    $('html, body').animate({
        scrollTop: $('#DIV_ID').offset().top - 20 //#DIV_ID is an example. Use the id of your destination on the page
    }, 'slow');
});

Java "user.dir" property - what exactly does it mean?

Typically this is the directory where your app (java) was started (working dir). "Typically" because it can be changed, eg when you run an app with Runtime.exec(String[] cmdarray, String[] envp, File dir)

Regular expressions in C: examples?

man regex.h reports there is no manual entry for regex.h, but man 3 regex gives you a page explaining the POSIX functions for pattern matching.
The same functions are described in The GNU C Library: Regular Expression Matching, which explains that the GNU C Library supports both the POSIX.2 interface and the interface the GNU C Library has had for many years.

For example, for an hypothetical program that prints which of the strings passed as argument match the pattern passed as first argument, you could use code similar to the following one.

#include <errno.h>
#include <regex.h>
#include <stdio.h>
#include <stdlib.h>
#include <string.h>

void print_regerror (int errcode, size_t length, regex_t *compiled);

int
main (int argc, char *argv[])
{
  regex_t regex;
  int result;

  if (argc < 3)
    {
      // The number of passed arguments is lower than the number of
      // expected arguments.
      fputs ("Missing command line arguments\n", stderr);
      return EXIT_FAILURE;
    }

  result = regcomp (&regex, argv[1], REG_EXTENDED);
  if (result)
    {
      // Any value different from 0 means it was not possible to 
      // compile the regular expression, either for memory problems
      // or problems with the regular expression syntax.
      if (result == REG_ESPACE)
        fprintf (stderr, "%s\n", strerror(ENOMEM));
      else
        fputs ("Syntax error in the regular expression passed as first argument\n", stderr);
      return EXIT_FAILURE;               
    }
  for (int i = 2; i < argc; i++)
    {
      result = regexec (&regex, argv[i], 0, NULL, 0);
      if (!result)
        {
          printf ("'%s' matches the regular expression\n", argv[i]);
        }
      else if (result == REG_NOMATCH)
        {
          printf ("'%s' doesn't the regular expression\n", argv[i]);
        }
      else
        {
          // The function returned an error; print the string 
          // describing it.
          // Get the size of the buffer required for the error message.
          size_t length = regerror (result, &regex, NULL, 0);
          print_regerror (result, length, &regex);       
          return EXIT_FAILURE;
        }
    }

  /* Free the memory allocated from regcomp(). */
  regfree (&regex);
  return EXIT_SUCCESS;
}

void
print_regerror (int errcode, size_t length, regex_t *compiled)
{
  char buffer[length];
  (void) regerror (errcode, compiled, buffer, length);
  fprintf(stderr, "Regex match failed: %s\n", buffer);
}

The last argument of regcomp() needs to be at least REG_EXTENDED, or the functions will use basic regular expressions, which means that (for example) you would need to use a\{3\} instead of a{3} used from extended regular expressions, which is probably what you expect to use.

POSIX.2 has also another function for wildcard matching: fnmatch(). It doesn't allow to compile the regular expression, or get the substrings matching a sub-expression, but it is very specific for checking when a filename match a wildcard (e.g. it uses the FNM_PATHNAME flag).

$.ajax( type: "POST" POST method to php

Id advice you to use a bit simplier method -

$.post('edit.php', {title: $('input[name="title"]').val() }, function(resp){
    alert(resp);
});

try this one, I just feels its syntax is simplier than the $.ajax's one...

How get an apostrophe in a string in javascript

You can put an apostrophe in a single quoted JavaScript string by escaping it with a backslash, like so:

theAnchorText = 'I\'m home';

How do I get first name and last name as whole name in a MYSQL query?

You can use a query to get the same:

SELECT CONCAT(FirstName , ' ' , MiddleName , ' ' , Lastname) AS Name FROM TableName;

Note: This query return if all columns have some value if anyone is null or empty then it will return null for all, means Name will return "NULL"

To avoid above we can use the IsNull keyword to get the same.

SELECT Concat(Ifnull(FirstName,' ') ,' ', Ifnull(MiddleName,' '),' ', Ifnull(Lastname,' ')) FROM TableName;

If anyone containing null value the ' ' (space) will add with next value.

Difference between ApiController and Controller in ASP.NET MVC

Use Controller to render your normal views. ApiController action only return data that is serialized and sent to the client.

here is the link

Quote:

Note If you have worked with ASP.NET MVC, then you are already familiar with controllers. They work similarly in Web API, but controllers in Web API derive from the ApiController class instead of Controller class. The first major difference you will notice is that actions on Web API controllers do not return views, they return data.

ApiControllers are specialized in returning data. For example, they take care of transparently serializing the data into the format requested by the client. Also, they follow a different routing scheme by default (as in: mapping URLs to actions), providing a REST-ful API by convention.

You could probably do anything using a Controller instead of an ApiController with the some(?) manual coding. In the end, both controllers build upon the ASP.NET foundation. But having a REST-ful API is such a common requirement today that WebAPI was created to simplify the implementation of a such an API.

It's fairly simple to decide between the two: if you're writing an HTML based web/internet/intranet application - maybe with the occasional AJAX call returning json here and there - stick with MVC/Controller. If you want to provide a data driven/REST-ful interface to a system, go with WebAPI. You can combine both, of course, having an ApiController cater AJAX calls from an MVC page.

To give a real world example: I'm currently working with an ERP system that provides a REST-ful API to its entities. For this API, WebAPI would be a good candidate. At the same time, the ERP system provides a highly AJAX-ified web application that you can use to create queries for the REST-ful API. The web application itself could be implemented as an MVC application, making use of the WebAPI to fetch meta-data etc.

Git error: src refspec master does not match any

You've created a new repository and added some files to the index, but you haven't created your first commit yet. After you've done:

 git add a_text_file.txt 

... do:

 git commit -m "Initial commit."

... and those errors should go away.

Add column to SQL Server

Adding a column using SSMS or ALTER TABLE .. ADD will not drop any existing data.

How to run a C# console application with the console hidden

If you're creating a program that doesn't require user input you could always just create it as a service. A service won't show any kind of UI.

Exposing a port on a live Docker container

I wrote a blog post that explains how to access an unpublished port of a container In different ways depending on the needs:

  • by committing a new image and running a new container,
  • by using socat to avoid restarting the container.

The post also goes through a brief introduction of both how port mapping works, the difference between exposing and publishing a port, and what is socat.

Here’s the link: https://lmcaraig.com/accessing-an-unpublished-port-of-a-running-docker-container

Where is body in a nodejs http.get response?

You need to add a listener to the request because node.js works asynchronous like that:

request.on('response', function (response) {
  response.on('data', function (chunk) {
    console.log('BODY: ' + chunk);
 });
});

What is the correct way to do a CSS Wrapper?

a "wrapper" is just a term for some element that encapsulates all other visual elements on the page. The body tag seems to fit the bill, but you would be at the mercy of the browser to determine what displays beneath that if you adjust the max-width.

Instead, we use div because it acts as a simple container that does not break. the main, header, footer, and section tags in HTML5 are just div elements named appropriately. It seems that there could (or should) be a wrapper tag because of this trend, but you may use whichever method of wrapping you find most suitable for your situation. through classes, ids and css, you can use a span tag in a very similar way.

There are a lot of HTML element tags that we do not use often or possibly even know about. Doing some research would show you what can be done with pure HTML.

Modifying CSS class property values on the fly with JavaScript / jQuery

Demo, IE demo

You could use the following function:

function setStyle(cssText) {
    var sheet = document.createElement('style');
    sheet.type = 'text/css';
    /* Optional */ window.customSheet = sheet;
    (document.head || document.getElementsByTagName('head')[0]).appendChild(sheet);
    return (setStyle = function(cssText, node) {
        if(!node || node.parentNode !== sheet)
            return sheet.appendChild(document.createTextNode(cssText));
        node.nodeValue = cssText;
        return node;
    })(cssText);
};

Features:

  • The function is written in vanilla-js, so it has better performance than jQuery alternatives
  • One stylesheet is created after the first call to setStyle, so if you don't call it, it won't create any stylesheet.
  • The same stylesheet is reused for the following calls of setStyle
  • The function return a reference to the node associated with the bunch of CSS that you have added. If you call the function again with that node as a second argument, it will replace the old CSS with the new one.

Example

var myCSS = setStyle('*{ color:red; }');
setStyle('*{ color:blue; }', myCSS); // Replaces the previous CSS with this one

Browser support

At least, it works on IE9, FF3, Chrome 1, Safari 4, Opera 10.5.

There's also an IE version which works both on modern browsers and old versions of IE! (Works on IE8 and IE7, but can crash IE6).

How to check if an email address exists without sending an email?

Other answers here discuss the various problems with trying to do this. I thought I'd show how you might try this in case you wanted to learn by doing it yourself.

You can connect to an mail server via telnet to ask whether an email address exists. Here's an example of testing an email address for stackoverflow.com:

C:\>nslookup -q=mx stackoverflow.com
Non-authoritative answer:
stackoverflow.com       MX preference = 40, mail exchanger = STACKOVERFLOW.COM.S9B2.PSMTP.com
stackoverflow.com       MX preference = 10, mail exchanger = STACKOVERFLOW.COM.S9A1.PSMTP.com
stackoverflow.com       MX preference = 20, mail exchanger = STACKOVERFLOW.COM.S9A2.PSMTP.com
stackoverflow.com       MX preference = 30, mail exchanger = STACKOVERFLOW.COM.S9B1.PSMTP.com

C:\>telnet STACKOVERFLOW.COM.S9A1.PSMTP.com 25
220 Postini ESMTP 213 y6_35_0c4 ready.  CA Business and Professions Code Section 17538.45 forbids use of this system for unsolicited electronic mail advertisements.

helo hi
250 Postini says hello back

mail from: <[email protected]>
250 Ok

rcpt to: <[email protected]>
550-5.1.1 The email account that you tried to reach does not exist. Please try
550-5.1.1 double-checking the recipient's email address for typos or
550-5.1.1 unnecessary spaces. Learn more at
550 5.1.1 http://mail.google.com/support/bin/answer.py?answer=6596 w41si3198459wfd.71

Lines prefixed with numeric codes are responses from the SMTP server. I added some blank lines to make it more readable.

Many mail servers will not return this information as a means to prevent against email address harvesting by spammers, so you cannot rely on this technique. However you may have some success at cleaning out some obviously bad email addresses by detecting invalid mail servers, or having recipient addresses rejected as above.

Note too that mail servers may blacklist you if you make too many requests of them.


In PHP I believe you can use fsockopen, fwrite and fread to perform the above steps programmatically:

$smtp_server = fsockopen("STACKOVERFLOW.COM.S9A1.PSMTP.com", 25, $errno, $errstr, 30);
fwrite($smtp_server, "helo hi\r\n");
fwrite($smtp_server, "mail from: <[email protected]>\r\n");
fwrite($smtp_server, "rcpt to: <[email protected]>\r\n");

Difference between HashMap and Map in Java..?

Map is an interface, i.e. an abstract "thing" that defines how something can be used. HashMap is an implementation of that interface.

How to check Grants Permissions at Run-Time?

Check out the below library in git :

RuntimePermission

Implementation :

        allprojects {
            repositories {
                ...
                maven { url 'https://jitpack.io' }
            }
        }
        
        dependencies {
            implementation 'com.github.manoj140220:RuntimePermission:1.0.3'
        }
    
    
      
    
        new RuntimePermission({Current Class Object}, String[] , {ActvityContext});
          
          String[] : permission array.
          example  : String[] permissionArray = {Manifest.permission.CAMERA, Manifest.permission.BODY_SENSORS,...} 
    
    Implement : {PermissionNotify}
    
    interface notifier methods.
    
    @Override
    public void notifyPermissionGrant() {
        
    }
    
    @Override
    public void notifyPermissionDeny() {
         
    }

adding css file with jquery

$('head').append('<link rel="stylesheet" href="style2.css" type="text/css" />');

This should work.

Electron: jQuery is not defined

Just install Jquery with following command.

npm install --save jquery

After that Please put belew line in js file which you want to use Jquery

let $ = require('jquery')

SecurityError: The operation is insecure - window.history.pushState()

We experienced the SecurityError: The operation is insecure when a user disabled their cookies prior to visiting our site, any subsequent XHR requests trying to use the session would obviously fail and cause this error.

RequiredIf Conditional Validation Attribute

I had the same problem yesterday, but I did it in a very clean way which works for both client side and server side validation.

Condition: Based on the value of other property in the model, you want to make another property required. Here is the code:

public class RequiredIfAttribute : RequiredAttribute
{
  private String PropertyName { get; set; }
  private Object DesiredValue { get; set; }

  public RequiredIfAttribute(String propertyName, Object desiredvalue)
  {
    PropertyName = propertyName;
    DesiredValue = desiredvalue;
  }

  protected override ValidationResult IsValid(object value, ValidationContext context)
  {
    Object instance = context.ObjectInstance;
    Type type = instance.GetType();
    Object proprtyvalue = type.GetProperty(PropertyName).GetValue(instance, null);
    if (proprtyvalue.ToString() == DesiredValue.ToString())
    {
      ValidationResult result = base.IsValid(value, context);
      return result;
    }
    return ValidationResult.Success;
  }
}

PropertyName is the property on which you want to make your condition
DesiredValue is the particular value of the PropertyName (property) for which your other property has to be validated for required

Say you have the following:

public enum UserType
{
  Admin,
  Regular
}

public class User
{
  public UserType UserType {get;set;}

  [RequiredIf("UserType",UserType.Admin,
              ErrorMessageResourceName="PasswordRequired", 
              ErrorMessageResourceType = typeof(ResourceString))]
  public string Password { get; set; }
}

At last but not the least, register adapter for your attribute so that it can do client side validation (I put it in global.asax, Application_Start)

DataAnnotationsModelValidatorProvider.RegisterAdapter(typeof(RequiredIfAttribute),
                                                      typeof(RequiredAttributeAdapter));

EDITED

Some people was complaining that the client side fires no matter what or it does not work. So I modified the above code to do conditional client side validation with Javascript as well. For this case you don't need to register adapter

 public class RequiredIfAttribute : ValidationAttribute, IClientValidatable
    {
        private String PropertyName { get; set; }
        private Object DesiredValue { get; set; }
        private readonly RequiredAttribute _innerAttribute;

        public RequiredIfAttribute(String propertyName, Object desiredvalue)
        {
            PropertyName = propertyName;
            DesiredValue = desiredvalue;
            _innerAttribute = new RequiredAttribute();
        }

        protected override ValidationResult IsValid(object value, ValidationContext context)
        {
            var dependentValue = context.ObjectInstance.GetType().GetProperty(PropertyName).GetValue(context.ObjectInstance, null);

            if (dependentValue.ToString() == DesiredValue.ToString())
            {
                if (!_innerAttribute.IsValid(value))
                {
                    return new ValidationResult(FormatErrorMessage(context.DisplayName), new[] { context.MemberName });
                }
            }
            return ValidationResult.Success;
        }

        public IEnumerable<ModelClientValidationRule> GetClientValidationRules(ModelMetadata metadata, ControllerContext context)
        {
            var rule = new ModelClientValidationRule
            {
                ErrorMessage = ErrorMessageString,
                ValidationType = "requiredif",
            };
            rule.ValidationParameters["dependentproperty"] = (context as ViewContext).ViewData.TemplateInfo.GetFullHtmlFieldId(PropertyName);
            rule.ValidationParameters["desiredvalue"] = DesiredValue is bool ? DesiredValue.ToString().ToLower() : DesiredValue;

            yield return rule;
        }
    }

And finally the javascript ( bundle it and renderit...put it in its own script file)

$.validator.unobtrusive.adapters.add('requiredif', ['dependentproperty', 'desiredvalue'], function (options) {
    options.rules['requiredif'] = options.params;
    options.messages['requiredif'] = options.message;
});

$.validator.addMethod('requiredif', function (value, element, parameters) {
    var desiredvalue = parameters.desiredvalue;
    desiredvalue = (desiredvalue == null ? '' : desiredvalue).toString();
    var controlType = $("input[id$='" + parameters.dependentproperty + "']").attr("type");
    var actualvalue = {}
    if (controlType == "checkbox" || controlType == "radio") {
        var control = $("input[id$='" + parameters.dependentproperty + "']:checked");
        actualvalue = control.val();
    } else {
        actualvalue = $("#" + parameters.dependentproperty).val();
    }
    if ($.trim(desiredvalue).toLowerCase() === $.trim(actualvalue).toLocaleLowerCase()) {
        var isValid = $.validator.methods.required.call(this, value, element, parameters);
        return isValid;
    }
    return true;
});

You need obviously the unobstrusive validate jQuery to be included as requirement

I want to exception handle 'list index out of range.'

for i in range (1, len(list))
    try:
        print (list[i])

    except ValueError:
        print("Error Value.")
    except indexError:
        print("Erorr index")
    except :
        print('error ')

How to force open links in Chrome not download them?

I think the question was about to open a local file directly instead of downloading a local file to the download folder and open the file in the download folder, which seems not possible in Chrome, except some add-on mentioned above.

My workaround would be to right click -> Copy the link location Windows + R and paste the link there and Enter It will go to the file directly.

Get cart item name, quantity all details woocommerce

This will show only Cart Items Count.

 global $woocommerce; 
    echo $woocommerce->cart->cart_contents_count;

How do I auto-hide placeholder text upon focus using css or jquery?

Sometimes you need SPECIFICITY to make sure your styles are applied with strongest factor id Thanks for @Rob Fletcher for his great answer, in our company we have used

So please consider adding styles prefixed with the id of the app container

_x000D_
_x000D_
    #app input:focus::-webkit-input-placeholder, #app  textarea:focus::-webkit-input-placeholder {_x000D_
        color: #FFFFFF;_x000D_
    }_x000D_
_x000D_
    #app input:focus:-moz-placeholder, #app textarea:focus:-moz-placeholder {_x000D_
        color: #FFFFFF;_x000D_
    }
_x000D_
_x000D_
_x000D_

Import PEM into Java Key Store

If you only want to import a certificate in PEM format into a keystore, keytool will do the job:

keytool -import -alias *alias* -keystore cacerts -file *cert.pem*

MySQL Cannot Add Foreign Key Constraint

Check the signing on both your table columns. If the referring table column is SIGNED, the referenced table column should be SIGNED too.

Adding padding to a tkinter widget only on one side

The padding options padx and pady of the grid and pack methods can take a 2-tuple that represent the left/right and top/bottom padding.

Here's an example:

import tkinter as tk

class MyApp():
    def __init__(self):
        self.root = tk.Tk()
        l1 = tk.Label(self.root, text="Hello")
        l2 = tk.Label(self.root, text="World")
        l1.grid(row=0, column=0, padx=(100, 10))
        l2.grid(row=1, column=0, padx=(10, 100)) 

app = MyApp()
app.root.mainloop()

'setInterval' vs 'setTimeout'

setTimeout(expression, timeout); runs the code/function once after the timeout.

setInterval(expression, timeout); runs the code/function in intervals, with the length of the timeout between them.

Example:

var intervalID = setInterval(alert, 1000); // Will alert every second.
// clearInterval(intervalID); // Will clear the timer.

setTimeout(alert, 1000); // Will alert once, after a second.

Handling exceptions from Java ExecutorService tasks

Instead of subclassing ThreadPoolExecutor, I would provide it with a ThreadFactory instance that creates new Threads and provides them with an UncaughtExceptionHandler

How can I create an error 404 in PHP?

What you're doing will work, and the browser will receive a 404 code. What it won't do is display the "not found" page that you might be expecting, e.g.:

Not Found

The requested URL /test.php was not found on this server.

That's because the web server doesn't send that page when PHP returns a 404 code (at least Apache doesn't). PHP is responsible for sending all its own output. So if you want a similar page, you'll have to send the HTML yourself, e.g.:

<?php
header($_SERVER["SERVER_PROTOCOL"]." 404 Not Found", true, 404);
include("notFound.php");
?>

You could configure Apache to use the same page for its own 404 messages, by putting this in httpd.conf:

ErrorDocument 404 /notFound.php

How do I implement __getattribute__ without an infinite recursion error?

Python language reference:

In order to avoid infinite recursion in this method, its implementation should always call the base class method with the same name to access any attributes it needs, for example, object.__getattribute__(self, name).

Meaning:

def __getattribute__(self,name):
    ...
        return self.__dict__[name]

You're calling for an attribute called __dict__. Because it's an attribute, __getattribute__ gets called in search for __dict__ which calls __getattribute__ which calls ... yada yada yada

return  object.__getattribute__(self, name)

Using the base classes __getattribute__ helps finding the real attribute.

Python logging not outputting anything

For anyone here that wants a super-simple answer: just set the level you want displayed. At the top of all my scripts I just put:

import logging
logging.basicConfig(level = logging.INFO)

Then to display anything at or above that level:

logging.info("Hi you just set your fleeb to level plumbus")

It is a hierarchical set of five levels so that logs will display at the level you set, or higher. So if you want to display an error you could use logging.error("The plumbus is broken").

The levels, in increasing order of severity, are DEBUG, INFO, WARNING, ERROR, and CRITICAL. The default setting is WARNING.

This is a good article containing this information expressed better than my answer:
https://www.digitalocean.com/community/tutorials/how-to-use-logging-in-python-3

Running stages in parallel with Jenkins workflow / pipeline

that syntax is now deprecated, you will get this error:

org.codehaus.groovy.control.MultipleCompilationErrorsException: startup failed:
WorkflowScript: 14: Expected a stage @ line 14, column 9.
       parallel firstTask: {
       ^

WorkflowScript: 14: Stage does not have a name @ line 14, column 9.
       parallel secondTask: {
       ^

2 errors

You should do something like:

stage("Parallel") {
    steps {
        parallel (
            "firstTask" : {
                //do some stuff
            },
            "secondTask" : {
                // Do some other stuff in parallel
            }
        )
    }
}

Just to add the use of node here, to distribute jobs across multiple build servers/ VMs:

pipeline {
  stages {
    stage("Work 1"){
     steps{
      parallel ( "Build common Library":   
            {
              node('<Label>'){
                  /// your stuff
                  }
            },

        "Build Utilities" : {
            node('<Label>'){
               /// your stuff
              }
           }
         )
    }
}

All VMs should be labelled as to use as a pool.

How to compile the finished C# project and then run outside Visual Studio?

On your project folder, open up the bin\Debug subfolder and you'll see the compiled result.

Changing width property of a :before css selector using JQuery

One option is to use an attribute on the image, and modify that using jQuery. Then take that value in CSS:

HTML (note I'm assuming .cloumn is a div but it could be anything):

<div class="column" bf-width=100 >
    <img src="..." />
</div>

jQuery:

// General use:
$('.column').attr('bf-width', 100);
// With your image, along the lines of:
$('.column').attr('bf-width', $('img').width());

And then in order to use that value in CSS:

.column:before {
    content: attr(data-content) 'px';
    /* ... */
}

This will grab the attribute value from .column, and apply it on the before.

Sources: CSS attr (note the examples with before), jQuery attr.

How do you write a migration to rename an ActiveRecord model and its table in Rails?

Here's an example:

class RenameOldTableToNewTable < ActiveRecord::Migration
  def self.up
    rename_table :old_table_name, :new_table_name
  end

  def self.down
    rename_table :new_table_name, :old_table_name
  end
end

I had to go and rename the model declaration file manually.

Edit:

In Rails 3.1 & 4, ActiveRecord::Migration::CommandRecorder knows how to reverse rename_table migrations, so you can do this:

class RenameOldTableToNewTable < ActiveRecord::Migration
  def change
    rename_table :old_table_name, :new_table_name
  end 
end

(You still have to go through and manually rename your files.)

Android Fatal signal 11 (SIGSEGV) at 0x636f7d89 (code=1). How can it be tracked down?

I got this error when I used ViewTreeObserver inside onDraw() function.

@Override
protected void onDraw(Canvas canvas) {
    // super.onDraw(canvas);
    ViewTreeObserver vto = mTextView.getViewTreeObserver();
    vto.addOnGlobalLayoutListener(new ViewTreeObserver.OnGlobalLayoutListener() {
        @Override
        public void onGlobalLayout() {
            // some animation        
        }
    });
 }

Android difference between Two Dates

I use this: send start and end date in millisecond

public int GetDifference(long start,long end){
    Calendar cal = Calendar.getInstance();
    cal.setTimeInMillis(start);
    int hour = cal.get(Calendar.HOUR_OF_DAY);
    int min = cal.get(Calendar.MINUTE);
    long t=(23-hour)*3600000+(59-min)*60000;

    t=start+t;

    int diff=0;
    if(end>t){
        diff=(int)((end-t)/ TimeUnit.DAYS.toMillis(1))+1;
    }

    return  diff;
}

sqldeveloper error message: Network adapter could not establish the connection error

Control Panel > Administrative Tools > Services >

Start OracleOraDb11g_home1TNSListener

UTF-8 encoded html pages show ? (questions marks) instead of characters

In my case, database returned latin1, when my browser expected utf8.

So for MySQLi I did:

 mysqli_set_charset($dblink, "utf8");    

See http://php.net/manual/en/mysqli.set-charset.php for more info

How can I copy columns from one sheet to another with VBA in Excel?

Selecting is often unnecessary. Try this

Sub OneCell()
    Sheets("Sheet2").range("B1:B3").value = Sheets("Sheet1").range("A1:A3").value
End Sub

Converting char* to float or double

You are missing an include : #include <stdlib.h>, so GCC creates an implicit declaration of atof and atod, leading to garbage values.

And the format specifier for double is %f, not %d (that is for integers).

#include <stdlib.h>
#include <stdio.h>

int main()
{
  char *test = "12.11";
  double temp = strtod(test,NULL);
  float ftemp = atof(test);
  printf("price: %f, %f",temp,ftemp);
  return 0;
}
/* Output */
price: 12.110000, 12.110000

md-table - How to update the column width

When Material creates the table it automagically applies two class-names for you which you can use to style each column. In the the example below the styles is named mat-column-userId and cdk-column-userId.

<ng-container cdkColumnDef="userId">
  <md-header-cell *cdkHeaderCellDef> ID </md-header-cell>
  <md-cell *cdkCellDef="let row"> {{row.id}} </md-cell>
</ng-container>

Now you can use those names in css:

.mat-column-userId {
  flex: 0 0 100px;
}

Similar to Rahul Patil's answer, but you don't need to add another class to your column definitions.

How do you create a Swift Date object?

According to Apple documentation

Example :

var myObject = NSDate()
let futureDate = myObject.dateByAddingTimeInterval(10)
let timeSinceNow = myObject.timeIntervalSinceNow

How to redirect the output of print to a TXT file

If you're on Python 2.5 or earlier, open the file and then use the file object in your redirection:

log = open("c:\\goat.txt", "w")
print >>log, "test"

If you're on Python 2.6 or 2.7, you can use print as a function:

from __future__ import print_function
log = open("c:\\goat.txt", "w")
print("test", file = log)

If you're on Python 3.0 or later, then you can omit the future import.

If you want to globally redirect your print statements, you can set sys.stdout:

import sys
sys.stdout = open("c:\\goat.txt", "w")
print ("test sys.stdout")